Haemopoietic System/Haematology/Blood/Blood Products Flashcards

1
Q

An 80-year-old man is planned for emergency surgery due to a strangulated direct hernia. He has been on clopidogrel after he sustained an acute myocardial infarction 10 months ago. Which one of the following is the next best in management prior to the surgery?
A. Give platelets and proceed to the surgery.
B. Give DDAVP and proceed to the surgery.
C. Give cryoprecipitate and proceed to the surgery.
D. Stop clopidogrel and proceed to the surgery.
E. Give vitamin K intravenously.

A

A. Give platelets and proceed to the surgery.

Strangulated hernias are genuine emergencies requiring immediate surgery. Failing to do so can lead to bowel perforation, peritonitis and death. This patient, however, is on clopidogrel and has an increased risk of intra- or post-operative bleeding due to effect of clopidogrel on platelet function. Since abdominal surgery is associated with marked risk of bleeding in this patient, platelets should be given prior to the surgery to counteract the effect of clopidogrel. If this patient had a coronary stent, antiplatelet therapy should have not been ceased. Exceptions to this include patients with coronary stents who are undergoing spinal, intracranial, extraocular TURP or major plastic reconstructive procedures. For these operations, patients at low risk of stent thrombosis should have their antiplatelet therapy routinely ceased perioperatively.

(Option B) Desmopressin (DDAVP), often in conjunction with fresh frozen plasma, is used for preoperative management of patients with Von-Willebrand disease in whom there is deficiency of factor VIII.

(Option C) Cryoprecipitate replenish coagulation factors with no effect on platelet activity inhibited by clopidogrel.

(Option D) It takes approximately 7 days for effects of antiplatelet drugs to suitably diminish. Stopping clopidogrel now will not decrease the chance of bleeding immediately and is not useful.

(Option E) Vitamin K is used for warfarin reversal and has no effect on platelet function.

TOPIC REVIEW
Coronary stent thrombosis is an uncommon but clinically devastating complication of coronary artery stenting that usually results in significant myocardial infarction or death. Approximately 40% of reported cases have occurred in the context of non-cardiac surgery (NCS) performed in patients with coronary artery stents, in whom dual antiplatelet therapy or clopidogrel alone has been ceased.

In patients with coronary disease, cessation of aspirin or clopidogrel is associated with an approximate 2-3 fold increase in subsequent death or myocardial infarction. This risk is further elevated in patients with intracoronary stent and is of added concern because the dramatic consequences of stent occlusion. There is uncertainty regarding the risks of stent thrombosis in individual patients, and in particular how to balance this risk against that of surgical complications if antiplatelet therapy is continued throughout the perioperative period.

The following are current recommendations regarding antiplatelet agents and non-cardiac surgery:
- Elective non-cardiac surgery should be deferred for at least 6 weeks and ideally 3 months following PCI with bare metal stenting.
- Elective surgery should be deferred for 12 months following drug eluting stents because of a likely increased risk of death/myocardial infarction/stent thrombosis.
- Despite the observation that dual antiplatelet therapy increases the likelihood of bleeding for most surgical procedures, the consequences of bleeding are less significant than those of stent thrombosis.
- The risk benefit ratio would favor continuation of aspirin in most patients and dual antiplatelet therapy in many patients with prior coronary artery stenting who are undergoing non-cardiac surgery.
- Exceptions to this include patients undergoing spinal, intracranial, extraocular TURP or major plastic reconstructive procedures. For these operations, patients at low risk of stent thrombosis should have their antiplatelet therapy routinely ceased perioperatively.
- In patients with coronary artery disease in whom NO stent has been placed, antiplatelets can be stopped 7 days before the surgery.
- In the event of emergency, surgery should be proceeded to without cessation of antiplatelet therapy. platelet transfusion might be considered in selected patients in anticipation or occurrence of major bleeding.
- In other situations than bare metal stents and drug eluting stents placed within the past 6 weeks and 12 months respectively, antiplatelet medications should be stopped 1 week (7 days) before the procedure.

** http://www.csanz.edu.au/wp-content/uploads/2014/12
http://www.racgp.org.au/afp/2014/december/new-oral

How well did you know this?
1
Not at all
2
3
4
5
Perfectly
2
Q

A 6-year-old girl with sickle cell disease is brought to your practice for routine follow-up. She is completely asymptomatic and has never had a sickle cell crisis. She takes no medications. Physical examination is unremarkable. Which one of the following complications is more likely to occur earliest in this patient?
A. Splenic infarction.
B. Bone infarction.
C. Splenic sequestration.
D. Stroke.
E. Acute coronary ischemia.

A

B. Bone infarction.

Vaso-occlusive phenomena and hemolysis are the clinical hallmarks of sickle cell disease (SCD) that is an inherited disorder due to homozygosity for the abnormal hemoglobin S (HBs). Vaso-occlusion results in recurrent painful episodes and a variety of serious organ system complications that can lead to lifelong disabilities and early death. Clinical signs and symptoms typically develop at an early age.

Bone infarction is the most common earliest presenting symptoms. This often presents as acute pain in the hands and/or feet, and is often the initial symptom occurring in 40% of all patients in general and 50% of children who become symptomatic before the age of two years. The cause is bone marrow infarction. Since pain tends to involve bones with highest bone marrow activity, and because marrow activity changes with age, different bone pain patterns are predictable. During the first 18 months of life, the metatarsal and metacarpals can be involved, presenting as dactylitis or hand-foot syndrome.

An acute episode of pain is the second most common initial presentation and the most common symptom after the age of two years.

Splenic sequestration is the third most common presenting symptom, occurring in 20% of patients overall and one-third of children before the age two years. Over time, vaso-occlusion can occur in virtually every organ system, accounting for the characteristic acute and chronic multisystem failure associated with the disease.

How well did you know this?
1
Not at all
2
3
4
5
Perfectly
3
Q

A 27-year-old man presents to the emergency department with pallor and jaundice. On physical examination, the spleen is palpated two cm below the costal margin. A blood exam reveals a reticulocyte count of 18% (normal 0-2%) and microspherocytosis. A Coomb’s test is positive. Which one of the following is the most likely cause of this clinical picture?
A. Acquired autoimmune hemolytic disease.
B. Hereditary spherocytosis.
C. Congenital autoimmune hemolytic disease.
D. Myeloproliferative disorder.
E. Glucose-6-phosphate dehydrogenase (G6PD) deficiency.

A

**A. Acquired autoimmune hemolytic disease **

The pallor and jaundice indicate the hemolytic nature of the process. With a palpable spleen and a high reticulocyte count, the process has been chronic and probably long-standing. With the positive Coombs test, the autoimmune nature of the condition is ensured with high certainty; therefore, of the options acquired autoimmune hemolytic disease is the most likely cause to consider. Microspherocytosis can be seen in cases of autoimmune hemolytic disease.

(Option B) Hereditary spherocytosis leads to episodes of jaundice and pallor caused by hemolytic anemia, and characteristic spherocytosis on peripheral smear; however, Coombs test is negative because the hemolysis is not autoimmune in nature.

Option C) Congenital autoimmune hemolytic disease would have presented much earlier in life.

(Option D) Myeloproliferative disorders are not associated with hemolysis.

(Option E) G6PD would have demonstrated its clinical features earlier in life. The condition is not autoimmune and Coombs test would be negative.

** Medscape - Hemolytic Anemia**

How well did you know this?
1
Not at all
2
3
4
5
Perfectly
4
Q

A group of college students are admitted to the emergency department with bruises over their skin, dark-colored urine and shock one week after they were back from camping in a forest. Which one of the following is most likely to be the cause of this presentation?
A. Giardia lamblia.
B. Escherichia coli.
C. Disseminated intravascular coagulopathy.
D. Staphylococcus aureus.
E. Ross River fever.

A

**B. Escherichia coli. **

The bruises over the skin (petechial rash), kidney involvement, and mental status alteration are consistent with thrombotic thrombocytopenic purpura (TTP) as the most likely diagnosis.

TTP is a rare blood disorder characterized by clotting in small blood vessels resulting in a low platelet count. In its full blown form, the disease consists of the pentad of (1) microangiopathic hemolytic anemia,
(2) thrombocytopenic purpura,
(3) neurological abnormalities,
(4) fever (non-infectious), and
(5) renal disease.
TTP can affect any organ system, but involvement of the peripheral blood, the central nervous system, and the kidneys causes the clinical manifestation.

Neurological manifestations include alteration in mental status, seizures, hemiplegia, paresthesias, visual disturbances, and aphasia. Patients may notice dark urine from hemoglobinuria. Severe bleeding from thrombocytopenia is unusual, but petechiae are common. Fever may occur in 50% of patients

E-coli O157:H7, E-coli O104:H4 and some other Shiga toxin-producing bacteria are the most common cause in children with TTP and hemolytic uraemic syndrome (HUS). In adults, many cases are idiopathic, but the same bacteria can cause these diseases as well. In adults immunosuppression and pregnancy can be other possible causes.

Since a group of persons are affected simultaneously, it is very likely that E-coli has been the etiology of the TTP.

(Option A) Giardia lamblia infection manifest with a different clinical picture including diarrhoea (non-bloody), flatulence, and crampy abdominal pain.

(Option C) DIC can present with similar picture (bleeding, shock, renal dysfunction, hepatic dysfunction, and central nervous system problems); however, the coincidence makes E-coli infection a more likely diagnosis.

(Option D) Staphylococcus aureus causes acute gastroenteritis with vomiting (more prominent) and diarrhea. Petechiae, neurological manifestations and shock are not present.

(Option E) Ross River fever is a mosquito-borne disease presenting with fever, rash, and polyarhtralgia.

*http://www.merckmanuals.com/professional/hematolog
*http://emedicine.medscape.com/article/206598-clini

How well did you know this?
1
Not at all
2
3
4
5
Perfectly
5
Q

A 70-year-old man from a low-level-of-care nursing home is brought to the hospital after he had a fall 3 hours ago. He is on warfarin for atrial fibrillation (AF). Laboratory studies show that he has an INR of 4.9. A CT scan of the head is ordered which is normal. Other investigations are unremarkable. Which one of the following is the next best step in management?
A. Stop warfarin.
B. Skip one dose of warfarin.
C. Give fresh frozen plasma (FFP).
D. Give vitamin K.
E. Repeat CT scan of the head in 2 days.

A

**B. Skip one dose of warfarin. **

The therapeutic range of warfarin for most patients varies between 2 to 3.5 (2-3, or occasionally 2.5-3.5).

Increased INR beyond therapeutic levels are associated with higher risk of bleeding. For patients with an INR above the therapeutic range but less than 5, who are not bleeding, skipping the next dose of warfarin and resuming lower doses once the INR is within the therapeutic range is the recommended management.

Ceasing warfarin, FFP, vitamin K (intravenously) and Prothrombinex are used in situations where there is active bleeding or the risk of bleeding is high.

*https://www.health.qld.gov.au/publications/clinica
*http://www.uptodate.com/contents/management-of-war
*https://www.mja.com.au/journal/2013/198/4/update-c

How well did you know this?
1
Not at all
2
3
4
5
Perfectly
6
Q

Which one of the following is the most common cause of anemia in geriatric population?
A. Iron deficiency anemia from blood loss.
B. Nutritional anemia from vitamin B12 deficiency.
C. Anemia of chronic disease.
D. Hemolytic anemia.
E. Myelodysplastic anemia.

A

**C. Anemia of chronic disease. **

Chronic diseases are the most common causes of anemia in the geriatric population accounting for 30-45% of cases. Iron deficiency is the second most common cause.

Cause: Percentage of cases
Chronic diseases 30-45%
Iron deficiency 15-30%
Post-hemorrhagic 5-10%
Vitamin B12/B9 deficiency 5-10% Chronic leukemia and lymphoma 5% Myelodysplastic syndromes 5%
No identifiable cause 15-25%

* AAFP - Anemia in the Elderly
* Evaluation and Management of Anemia in the Elderly

How well did you know this?
1
Not at all
2
3
4
5
Perfectly
7
Q

A 32-year-old African male presents to the Emergency Department with fatigue, jaundice, dark-colored urine and acute decrease in hemoglobin for the past four days. Liver function tests are normal, except for an elevated unconjugated bilirubin level. Which one of the laboratory results would be most consistent with the diagnosis of hemolysis due to glucose-6-phosphate dehydrogenase deficiency?
A. Spherocytosis.
B. Schistocytosis.
C. Positive Coomb’s test.
D. Heinz bodies.
E. Elliptocytosis.

A

**D. Heinz bodies. **

Glucose-6-phosphate dehydrogenase (G6PD) deficiency is the most common enzyme deficiency in human, affecting 400 million people worldwide, with a high prevalence in people of African, Asian, and Mediterranean descent. The condition is inherited as an X-linked recessive disorder. It is polymorphic with more than 300 variants.

G6PD deficiency can present as neonatal hyperbilirubinemia. People with this disorder can experience episodes of brisk hemolysis in response to oxidative stresses or, less commonly, have chronic hemolysis. However, many individuals with G6PD deficiency are asymptomatic.
G6PD deficiency confers partial protection against malaria.

The diagnosis of G6PD deficiency can be made on the basis of a well-documented history, evidence of hemolysis, a peripheral-blood smear showing Heinz bodies (erythrocytes with denatured hemoglobin) and ‘bite cells’. Measurement of G6PD activity while the patient is in remission is the gold-standard diagnostic test for G6PD deficiency.

G6PD deficiency should be suspected in all patients with non-immune acute hemolysis and no spherocytosis on laboratory testing.

(Option A) Spherocytes are characteristic feature in hereditary spherocytosis.

(Option B) Schistocytes are seen in microangiopathic hemolytic anemia (MAHA) associated with hemolytic uremic syndrome and thrombotic thrombocytopenic purpura (TTP).

(Option C) Positive Coombs test is the characteristic feature of immune-mediates hemolytic anemia.

(Option E) Elliptocytosis, also known and ovalocytosis, is a feature of hereditary elliptocytosis – an inherited disorder in which an abnormally large number of patient’s red blood cells are elliptical rather than the typical biconcave disc-shaped.

* UpToDate - Diagnosis and management of glucose-6-phosphate dehydrogenase (G6PD) deficiency
* Medscape - Glucose-6-Phosphate Dehydrogenase (G6PD) Deficiency

How well did you know this?
1
Not at all
2
3
4
5
Perfectly
8
Q

A 32-year-old woman, accompanied by his brother, presents with complaint that every time she gets a cut, it takes a long for the bleeding to stop. She also mentions that she is distressed with her heavy periods, for which she has to use 10 pads a day. Her brother mentions that she has been like that since he can remember. She is otherwise healthy with no significant finding on physical examination. Which one of the following is the most likely diagnosis?
A. Hemophilia A.
B. Stuart disease.
C. Von Willebrand disease.
D. Factor IX deficiency.
E. Immune thrombocytopenic purpura (ITP).

A

**C. Von Willebrand disease. **

Long-standing history of prolonged bleeding after trauma and history of menorrhagea in an otherwise healthy woman is highly suggestive of Von Willebrand disease as the most likely cause.

Von Willebrand disease (VWD) is an inherited disease with many different types (22 types) and clinical pictures. Almost all types cause mild bleeding problems with excellent prognosis. The most common types often have an autosomal dominant inheritance. This disease is very common (1 in 100 population) and is the most common inherited bleeding disorder.

Von Willebrand factor is a circulating factor that is attached to factor VIII. This factor by gluing platelets together and to the vascular lining plays the earliest role in coagulation. Because the clinical disease can be very mild, most cases will never be diagnosed, but if symptomatic, the symptoms can include:
- Easy bruising
- Mucosal bleeding (e.g. epistaxis, menorrhagoea, gastrointestinal bleeding, etc)
- No history of hemarthroses or intramuscular hematomas (except for type 3 that is very rare but can have musculoskeletal manifestations)
- Prolonged bleeding after trauma or surgery

(Option A) Hemophilia A presents with factor type of bleeding (deep and delayed). As both haemophilia A and B are X-linked, women can all be carriers. Homozygous females never born.

(Option B) Stuart disease or factor X (Stuart-Prower factor) deficiency is one of the world’s rarest factor deficiencies. Factor X is a vitamin K-dependent factor that serves as the first enzyme in the common pathway of thrombus formation. It can cause both platelet and factor types of bleeding.

(Option D) Like hemophilia A, factor IX deficiency (haemophilia B) is never seen in an alive woman.

(Option E) ITP presents with petechiae and bruising rather than prolonged bleeding. Furthermore, compared to VWD, chronic ITP in adults is less common.

How well did you know this?
1
Not at all
2
3
4
5
Perfectly
9
Q

A 27-year-old man presents with sudden onset of jaundice, pallor, and dark urine three days after taking of primaquine for malaria. On blood tests, hemoglobin is 52 g/L (120-160g/L) and unconjugated bilirubin is elevated. Blood film shows ‘bite cells’. Which one of the following is the most likely diagnosis?
A. Iron deficiency anemia.
B. Glucose-6-phosphate dehydrogenase deficiency.
C. Anemia due to blood loss.
D. Acute hepatitis.
E. Chronic renal failure.

A

**B. Glucose-6-phosphate dehydrogenase deficiency. **

Development of jaundice after use of primaquine is suggestive of glucose-6-phosphate dehydrogenase (G6PD) deficiency.

The typical presentation is sudden onset of jaundice, pallor, and dark urine, with or without abdominal and back pain. This is associated with an abrupt fall in the hemoglobin concentration of 30-40 g/L during which time the peripheral blood smear reveals red cell fragments, microspherocytes, and eccentrocytes or “bite” cells.

Special stains show** Heinz bodies**, which are collections of denatured globin chains often attached to the red cell membrane.

Episodes of acute hemolytic anemia are triggered by oxidative stress. Infections are the most common triggering factor.

Iron deficiency anemia, anemia due to blood loss and anemia of chronic renal failure are not associated with elevated unconjugated bilirubin. This fining is a feature of hemolytic anemia.

* UpToDate - Diagnosis and management of glucose-6-phosphate dehydrogenase (G6PD) deficiency
* Medscape - Glucose-6-Phosphate Dehydrogenase (G6PD) Deficiency

How well did you know this?
1
Not at all
2
3
4
5
Perfectly
10
Q

A 23-year-old African man presents to your clinic with pallor and dark urine after he took cotrimoxazole for an acute respiratory infection. Blood investigations are significant for hemoglobin of 90 g/L and a reticulocyte count of 5%. Both direct and indirect Coombs tests are negative. Serum electrophoresis shows type A hemoglobin. There is no family history of such presentation. Which one of the following is the most likely diagnosis?
A. Glucose-6-phosphate dehydrogenase deficiency.
B. Autoimmune hemolytic anemia.
C. Sickle cell anemia.
D. Thalassemia.
E. Hereditary spherocytosis.

A

A. Glucose-6-phosphate dehydrogenase deficiency.

Of the options, glucose-6-phosphate (G6PD) deficiency is the most likely diagnosis.

G6PD deficiency is an X-linked enzymatic defect that can cause hemolysis after oxidative stresses such as acute illness or ingestion of specific drugs. The most commonly implicated drugs are sulfa drugs (e.g., cotrimoxazole), primiquine, dapsone, quinidine and nitrofurantoin. The most common type of oxidant stress, however, is infections not drugs.

Type-A G6PD deficiency is the milder form and is more common among black people, whereas type B is mostly seen in patients of north Mediterranean origin. Type B has a more severe presentation.

The usual presentation is similar to all hemolytic anemias i.e. low hemoglobin, high LDH, decreased haptoglobin, elevated bilirubin (mostly indirect) and increased reticulocyte count in the presence of a normal MCV. An elevated reticulocyte count is a criterion for hemolysis but is not specific for it.

(Option B) Negative Coombs test rules out the possibility of autoimmune hemolytic anemia.

Options C and D) With normal hemoglobin (hemoglobin A, which includes two normal α chains and two normal β chains) on serum electrophoresis, sickle cell disease and thalassemia are excluded.

(Option E) With a negative family history of hereditary spherocytosis, this disease is less likely to be the cause of hemolysis in this patient. Another clue against such diagnosis is that hereditary spherocytosis manifestation occur much earlier in life.

* UpToDate - Diagnosis and management of glucose-6-phosphate dehydrogenase (G6PD) deficiency
* Medscape - Glucose-6-Phosphate Dehydrogenase (G6PD) Deficiency

How well did you know this?
1
Not at all
2
3
4
5
Perfectly
11
Q

A 23-year-old woman presents to your practice with complaint of breathlessness while climbing stairs. On blood tests, she has hemoglobin of 7 g/dL. Which one of the following is the most appropriate next step in management?
A. Transfusion of packed cells.
B. Colonoscopy.
C. Small bowel biopsy.
D. Parenteral iron.
E. Fecal occult blood test (FOBT).

A

A. Transfusion of packed cells.

According to guidelines, transfusion of packed red cells is not indicated for those with Hb >10 g/dL. The lower threshold varies from 6 g/dL to 8 g/dL; however, hemoglobin level alone cannot be used to determine the need for packed cell transfusion.

It is recommended that symptomatic anemia be treated with transfusion of packed cells in all patients with Hb<10 g/dL, regardless of the Hb level, provided that the symptoms are severe enough and are clearly related to the anemia rather than the underlying condition. This is true regardless of the underlying etiology of the anemia.

Colonoscopy (option B), FOBT (option E), or small biopsy (option C) may be indicated as attempts to spot the source or iron loss or deficiency where iron deficiency is the underlying cause of anemia. Before proceeding to such tests, iron deficiency should be diagnosed by iron studies. Iron replacement (most commonly by oral supplementation) is the treatment of established iron deficiency anemia. Parentral iron (option D) may be used in very severe cases of iron deficiency anemia.

NOTE – According to most references, exertion-related symptoms of anemia are not indications for transfusion of packed cells. In this question, however, the only acceptable option will be transfusion of packed cells, as no other option is correct regarding this clinical scenario.

* MJA - Diagnosis and management of iron deficiency anaemia: a clinical update * NCBI - Blood Transfusion - Recommendations for the transfusion of red blood cells

How well did you know this?
1
Not at all
2
3
4
5
Perfectly
12
Q

Accompanied by his wife, a 68-year-old man is brought to the emergency department. The wife mentions that her husband woke up with a nagging headache. One hour later, he started to develop slurred speech and weakness of the right arm. She mentioned that her husband has been on a pill for his cardiac condition for the past 3 months, but she cannot remember the name. Since his pulse seems irregular, you assume that the pill is warfarin and check an INR which comes back 4.5. A non- contrast CT scan of the head reveals intracranial hemorrhage. Which one of the following is the next best action to take?
A. Give fresh frozen plasma (FFP).
B. Give FFP and vitamin K.
C. Give protamine sulfate.
D. Give vitamin K.
E. Switch to unfractionated heparin.

managing patients on warfarin therapy with bleeding
A

**B. Give FFP and vitamin K. **

Bleeding is the most common complication of warfarin therapy and is related to the INR value. Although incremental rises in INR increase the risk of bleeding, most intracranial bleedings occurs in patients with an INR in the therapeutic range. Such events occur in 0.5-1.0% of patients with AF per year.

In the event of major or life-threatening bleeding associated with warfarin use, prothrombin complex concentrate (PCC) with or without FFP is the most appropriate next step in management. FFP in large volumes should be used if PCC is not available. In major or life-threatening bleedings, 5-10mg of vitamin K should also be given in conjunction with PCC or FFP to sustain the reversal effect. Cessation of warfarin should be considered in all patients with bleeding; however, cessation of warfarin alone is not going to change the immediate management plan that is reversing its effects immediately.

(Option A) FFP alone immediately reverses the warfarin effect, but vitamin K is aslo necessary to sustain the response after the effect of FFP wears out.

(Option C) Protamine sulfate is the antidote to heparin with no effect on warfarin.

(Option D) Vitamin K is an effective antidote to the anticoagulation effect of warfarin. Despite this, data are lacking to show that its use improves outcome in life-threatening bleeding. The usual dose is 5-10mg administered orally or intravenously. Intravenous route achieves a more rapid response compared with oral administration, with an onset of action between 6-8 hours. However, both routes achieve a similar correction of the INR by 24 hours. Vitamin K is the treatment of choice if the goal is to normalize the INR and no immediate counteraction against major organ bleeding or life-threatening bleeding is prompted. Intravenous administration is the preferred method.

(Option E) Switching to heparin does not decrease the risk of bleeding; nor does it counteract warfarin effects.

TOPIC REVIEW
Prothrombin complex concentrate (PCC) comes in two forms. One formulated with three factors (II, IX and X) and the other with four factors (II, VII, IX and X). Advantages of PCC over FFP include rapid reconstitution into a small volume for infusion over 20–30 minutes, fast onset of action (10-15 minutes), no requirement to check a patient’s blood group, minimal risk of viral transmission due to pathogen reduction and inactivation steps during manufacturing, and reduced risk of other clinical adverse reactions such as transfusion-associated circulatory overload or transfusion-associated acute lung injury. Prothrombinex-VF, a three-factor PCC, is the only product currently in routine use in Australia and New Zealand for warfarin reversal. Despite instructions by the Warfarin Reversal Consensus Guidelines published in 2004 recommended that it be supplemented with FFP, there have been several reports of successful use of Prothrombinex-VF without addition of FFP. Prothrombinex-VF has been used successfully to electively reverse anticoagulation in patients on warfarin therapy with a stable INR, and achieved the target INR in over 90% of patients. However, since the efficacy of Prothrombinex-VF alone has not been extensively evaluated for patient with major or life-threatening bleeding or INR > 10, current recommendation is that Prothrombinex-VF is supplemented with FFP for addition of adequate amounts of factor VII to ensure optimal reversal of the anticoagulant effect of warfarin when major or life-threatening bleeding is a concern.

For life-threatening (critical organ) and clinically significant bleeds, the consensus is to use the maximum dose of Prothrombinex-F (with vitamin K1 and FFP) and the maximum amount of FFP when Prothrombinex-VF is unavailable.

Recommendations for managing patients on warfarin therapy with bleeding are summarized in the following table:

(TABLE) (Attached with question)

Recommendations for managing patients on warfarin therapy with a high INR but no bleeding are summarized in the following table:

(TABLE) (See photo below)

https://www.mja.com.au/journal/2013/198/4/update-c

How well did you know this?
1
Not at all
2
3
4
5
Perfectly
13
Q

A 67-year-old man in brought to the emergency department with sudden-onset severe headache and confusion. He was diagnosed with atrial fibrillation (AF) 4 months ago, for which he has been on warfarin since then. Blood studies show an INR of 3.5. A CT scan of the head reveals intracerebral hemorrhage. Which one of the following is the most appropriate next step in management?
A. Stop warfarin.
B. Vitamin K.
C. Fresh frozen plasma (FFP).
D. Increase the dose of warfarin.
E. Reduce the dose of warfarin.

A

**C. Fresh frozen plasma (FFP). **

Bleeding is the most common complication of warfarin therapy and is related to the INR value. Although the risk of hemorrhage is directly related to the INR value, most events of intracranial hemorrhage occur in patients with an INR within the therapeutic range. Such events occur in 0.5-1.0% of patients with AF per year.

In the event of major or life-threatening bleeding associated with warfarin use, prothrombin complex concentrate (PCC) with or without FFP is the most appropriate next step in management. FFP in large volumes should be used if PCC is not available.

(Option A) Cessation of warfarin should be considered in all patients with bleeding; however, cessation of warfarin alone is not going to change the immediate manage plan that is reversing its effects immediately.

(Option B) Vitamin K is an effective antidote for the anticoagulation effect of warfarin. Despite this, data are lacking to show that its use improves outcome in life-threatening bleedings. The usual dose is 5-10mg administered orally or intravenously. Intravenous route achieves a more rapid response compared with oral administration, with an onset of action between 6-8 hours. However, both routes achieve a similar correction of INR by 24 hours. In major or life-threatening bleeds, 5-10mg of vitamin K should be given in conjunction with PCC or FFP to sustain the reversal effect. Vitamin K is the treatment of choice for patients on warfarin therapy with bleeding in whom the aim is to normalize the INR (not immediate counteraction against major organ bleeding or life-threatening bleeding), vitamin K given intravenously is the preferred treatment.

(Option D) Increasing the dose of warfarin worsens the condition and not a correct option. Reducing the dose of warfarin can lead to decreased INR in long-term. It does not counteract the bleeding.

(Option E) While the patient is bleeding, dose reduction of the warfarin is not an appropriate option. Dose reduction is an option for patients with INRs beyond the target but no active bleeding.

* MJA - An update of consensus guidelines for warfarin reversal

How well did you know this?
1
Not at all
2
3
4
5
Perfectly
14
Q

A 73-year-old man presents to the emergency department with fracture of the right femoral neck after he sustained a fall at home. He underwent coronary artery drug eluting stent placement 2 months ago, and has been on clopidogrel since then. A full blood count (FBC) is normal; however, he has several bruises over his body. He requires surgery for fixation of the fracture. Which one of the following is the most appropriate management of this patient?
A. Stop clopidogrel and proceed with surgery in one week.
B. Do the surgery now.
C. Administer fresh frozen (FFP) plasma and proceed with the surgery.
D. Give platelets and proceed with the surgery.
E. Switch to heparin and perform the surgery in 7 days.

A

**B. Do the surgery now. **

Femoral neck fracture requires emergency surgical fixation, because other measures such as traction or rest pose a more significant risk to the patient than does the risk of bleeding associated with antiplatelet therapy. Patients for whom surgery cannot be deferred should be operated on immediately. Any option suggesting deferral of the surgery is inappropriate. On the other hand, this patient has undergone coronary stenting with drug-eluting stent and requires at least 12 months of antiplatelet therapy. Cessation of antiplatelet therapy is associated with significantly increased risk of stent thromobosis and grave complications; hence antiplatelet therapy should be continued. He should undergo emergecny surgery while he is on clopidogrel.

Of note, an Australian retrospective review of 181 patients with proximal femoral fracture demonstrated no significant difference in the amount of bleeding, transfusion requirement, complications rate, or length of stay in 16 patients taking clopidogrel and in 48 taking aspirin compared to others.

(Options A and E) Deferral of an emergency surgery is incorrect.

(Option C) FFP reverses the effect of heparin and warfarin. It has no effect on inhibited action of platelets.

(Option D) Platelet transfusion might be considered in selected patients in whom the risk of major bleeding clearly outweighs the benefits of counteracting the antiplatelet therapy. However, transfusion of platelets in this patient may result in stent thrombosis and is not advisable. The only exception is when the risk of bleeding is so high that remarkably outweighs the risk of stent thrombosis.

* http://www.csanz.edu.au/wp-content/uploads/2014/12
*http://www.aafp.org/afp/2010/1215/p1484.html
*http://www.racgp.org.au/afp/2014/december/new-oral
*http://bja.oxfordjournals.org/content/111/suppl_1/

How well did you know this?
1
Not at all
2
3
4
5
Perfectly
15
Q

A 66-year-old man presents with altered bowel habits, decreased stool caliber and rectal bleeding in the form of blood covering the stool. Investigations show that he has a colorectal cancer. He is planned for surgical tumor resection. Currently, he is on warfarin due to atrial fibrillation (AF) and has an INR of 2.5. Which one of the following is the most appropriate option to consider for warfarin reversal prior to the surgery?
A. Proceed with the surgery.
B. Give vitamin K before the surgery. C. Give fresh frozen plasma (FFP) and proceed with the surgery.
D. Wait for 3 months.
E. Switch to clopidogrel and perform the surgery in one week.

A

**B. Give vitamin K before the surgery. **

This patient, as a candidate for a surgical procedure, is at increased risk of intra- or post-operative bleeding due to warfarin therapy. Resection of a colonic tumor is an elective procedure that allows for planned action for warfarin reversal. In those surgical candidates with an INR of 2-3, cessation of warfarin 4-5 days before the surgery and administration of vitamin K (3mg, intravenously) the evening before the surgery is the preferred plan.

(Option A) Proceeding with a major surgery without warfarin reversal is associated with a significantly increased risk of bleeding and not appropriate.

(Option C) FFP was the correct answer if this patient required emergency surgery. Prothrombinex-VF (or if unavailable FFP) is used for rapid reversal of warfarin effect (in minutes) for urgent surgical procedures associated with increased risk of bleeding. Using FFP for warfarin reversal in elective procedures is not appropriate.

(Option D) Deferring the surgery for 3 months is unnecessary and inappropriate because warfarin reversal can be achieved in 5 days, maximum. There is also no other medical condition to preclude surgery in this patient.

(Option E) Clopidogrel is an antiplatelet medication. Switching to clopidogrel not only does not counteract the effect of warfarin, but it also adds to the risk of bleeding by inhibiting platelet aggregation.

https://www.mja.com.au/journal/2013/198/4/update-c

How well did you know this?
1
Not at all
2
3
4
5
Perfectly
16
Q

A 22-year-old woman comes to the emergency department complaining of epistaxis for the past 12 hours. She denies any trauma to her nose and face. On examination, she is noted to have several bruises over her chest, arms and legs and petechiae on the inner side of her lower lip. Laboratory tests revealed a platelet count of 223000/mm3, a PT of 12 seconds, and a prolonged APTT of 57 seconds. Her blood tests are otherwise normal. Which one of the following could be the most likely diagnosis?
A. Hemophilia.
B. Von Willebrand disease.
C. Factor XII deficiency.
D. Hemolytic uremic syndrome (HUS).
E. Idiopathic thrombocytopenic purpura (TTP).

A

**B. Von Willebrand disease. **

On approach to a bleeding disorder, the first thing to consider is evaluating whether it is platelet versus factor type of bleeding.

  • Platelet type of bleeding is superficial and causes gingival bleeding, epistaxis, ecchymoses, petechiae and purpura.
  • Factor type of bleeding occurs more deeply in joints or muscles causing hematomas.
  • Gastrointestinal, genitourinary, or central nervous system bleeding could be caused by both types.

This patient has platelet type of bleeding in the presence of a normal platelet count, a normal PT and prolonged ATTP. The clinical and laboratory picture is highly suggestive of Von Willebrand disease as the most likely diagnosis.

Von Willebrand disease is the most common hereditary bleeding disorder affecting 1% of population and characterized by a defect in production of Von Willebrand factor (VIIIa). Of all affected persons, 1% may become symptomatic at any age. Von Willebrand presents with superficial bleeding tendency in the presence of normal platelet count and PT, but an often elevated APTT. The reason for increased ATTP is the fact that defect in Von Willebrand factor results in decreased factor VIII level of activity.

(Option A) Hemophilia is only expressed in male individuals. Females can only carry the gene, as the pattern of inheritance is X-linked recessive.

(Option C) Factor XII deficiency is a very rare genetic disease which never causes bleeding. Hemolytic uremic syndrome is characterised by the presence the helmet cells (schistocytes or fragmented re cells) on the peripheral blood smear with features of haemolytic anemia such as decreased red cell counts and hemoglobin, reticulocytosis, elevated LDH, and probably jaundice.

(Option D) The primary clot formation and homeostasis is the function of platelets. After endothelial lining is damaged, Factor VIII and Von Willebrand factor are released from underneath the epithelial cells and cause the platelets not adhere to each and to the endothelial lining. Several hours later (after the fibrin clot is formed) a metaloprotease (ADAMTS13) dissolves the Von Willebrand factor to let the blood stream washes the platelets away. The deficiency of this enzyme leads to hemolytic uremic syndrome (HUS) and thrombotic thrombocytopenic purpura (TTP). A microangiopathic hemolytic anemia is the essence of the diagnosis. This patient does not have hemolytic anemia.

(Option E) Immune thrombocytopenia (ITP) typically occurs in a young otherwise healthy patient and is characterized by isolated fall in platelet count and platelet type of bleeding.

How well did you know this?
1
Not at all
2
3
4
5
Perfectly
17
Q

A 56-year-old woman is brought to the emergency department with sudden-onset severe epigastric pain. On examination, she has a blood pressure of 90/55mmHg, heart rate of 110bpm, and respiratory rate of 22 breaths per minute. There is abdominal guarding and rigidity, as well as marked tenderness and rebound tenderness over the epigastric area. A chest X-ray reveals free air under the right hemi-diaphragm. Based on clinical findings, a perforated peptic ulcer is diagnosed and the patient is planned for emergency laparotomy. She is on warfarin due to deep vein thrombosis (DVT) that developed 2 weeks ago. Which one of the following is the most appropriate next step in management?

A. Stop warfarin, give vitamin K and do the surgery.
B. Proceed with the surgery.
C. Give fresh frozen plasma (FFP) and proceed with the surgery.
D. Add intravenous heparin and perform the surgery.
E. Stop warfarin, give her heparin and proceed with the surgery.

A

**C. Give fresh frozen plasma (FFP) and proceed with the surgery. **

This patient is in need of emergency life-saving surgery in the setting of perforated peptic ulcer disease and clinical manifestation of peritonitis (abdominal rigidity and guarding). In the event of need for life-saving or emergency surgeries where deferring the surgery is not possible, warfarin effect should be reversed immediately using prothrombin complex concentrate (CCP) (Prothrombinex-VF®), or fresh frozen plasma (FFP) if CCP unavailable, regardless of the risk of potential VTE if warfarin is temporarily stopped. Any option offering addition of vitamin K to cessation of warfarin and administration of CCP (or FFP) would be the most appropriate management (not an option).

(Option A) Cessation of warfarin and administration of vitamin K was the method of choice if the patient had an INR of 2-3 within the past 2-4 weeks and there was a time window for deferral of the surgery for at least 24 hours. In such cases, vitamin K is given intravenously the evening before the surgery and INR is checked on the day of the surgery. An INR of ≤1.5 is safe to proceed with the surgery. If INR is >1.5 Prothrombinex-VF® should be given (FFP is used if Prothrombinex-VF is not available)

(Option B) Proceeding with the surgery without warfarin reversal is associated with significant risk of intra- or post-operative bleeding and not recommended.

(Option D) Addition of heparin adds to the risk of bleeding.

(Option E) Cessation of warfarin and bridging with heparin is indicated in patients with high risk of VTE, who are planned for elective surgery. The protocol is not used for emergency procedures.

TOPIC REVIEW
Management of patients on warfarin therapy who are undergoing an invasive procedure is according to the following table:

How well did you know this?
1
Not at all
2
3
4
5
Perfectly
18
Q

A 55-year-old man has been on warfarin for AF for the past 3 months. He presented with an incarcerated inguinal hernia and was booked for emergent surgery. Warfarin was stopped and fresh frozen plasma was given. Which one of the following is the time to resume warfarin therapy?
A. 12 hours post-op.
B. 48 hours post-op.
C. Immediately after recovery from anesthesia.
D. 5 days post-op.
E. When INR is less than 1.8 again.

A

**A. 12 hours post-op. **

For those, whose warfarin therapy has been stopped before major surgical procedures, it is recommended that the previous maintenance dose of warfarin be resumed on the night of surgery (12-24 hours).

In addition to warfarin, low molecular weight heparin (LMWH) in prophylactic dose or unfractionated heparin (UFH) with slow infusion is started at the same time. The target APTT is 1.5 times the normal. LMWH or UFH is continued for at least 5 days and is ceased 48 after the target INR is reached (≥1.8).

References * https://www.mja.com.au/journal/2013/198/4/update-c * https://www.seslhd.health.nsw.gov.au/rhw/manuals/d

How well did you know this?
1
Not at all
2
3
4
5
Perfectly
19
Q

A 65-year-old woman underwent an emergency surgery for a strangulated femoral hernia 2 days ago. She was on warfarin that was ceased before the surgery and fresh frozen plasma was given. Which one of the following is the most appropriate action to take regarding resuming anticoagulation?
A. Start the patient on LMVH.
B. Start the patient on unfractionated heparin.
C. Resume warfarin.
D. Resume warfarin and start LMWH. E. Resume warfarin after one week.

A

D. Resume warfarin and start LMWH.

For patients whose warfarin therapy has been stopped before major surgical procedures, it is recommended that the previous maintenance dose of warfarin be resumed on the night of surgery (12-24 hours) and prophylactic dose of LMWH started at the same time.

Unfractionated heparin (UFH) can substitute LMWH as a second-line option. Unlike LMWH, UFH use requires monitoring to a target APTT of 1.5 times the normal. If unfractionated heparin (UFH) is used bolus injection should be avoided and slow infusion used. LMWH or UFH is continued for at least 5 days and is ceased 48 after the target INR is reached (≥1.8). The reason for co-administration of LMWH or UFH is the fact that warfarin initially increases the risk of thromboembolism.

References * https://www.mja.com.au/journal/2013/198/4/update-c * https://www.seslhd.health.nsw.gov.au/rhw/manuals/d

How well did you know this?
1
Not at all
2
3
4
5
Perfectly
20
Q

Which one of the following conditions is the most common inherited hypercoagulable state leading to venous thromboembolism (VTE)?
A. Anti-thrombin deficiency.
B. Protein C deficiency.
C. Protein S deficiency.
D. Factor V Leiden.
E. Lupus anticoagulant.

A

**D. Factor V Leiden. **

Venous thromboembolism (VTE) is a multifactorial disease driven by environmental/acquired risk factors such as age, obesity, oral contraceptives, and immobility, as well as inherited risk factors such as genetic polymorphisms.

Of genetic factors, factor V Leiden is the most common genetic cause of VTE . Prothrombin G20210A, gene mutation and deficiencies in protein S, protein C, and anti-thrombin account for most of the remaining cases.

It should be noted however that although factor V Leiden is the most common genetic cause of VTE, it is associated with only a medium risk of VTE in carriers.

References * MedScape - Genetics of Venous Thromboembolism * PubMed - Venous Thromboembolism: Classification, Risk Factors, Diagnosis, and Management

How well did you know this?
1
Not at all
2
3
4
5
Perfectly
20
Q

Which one of the following hereditary conditions is associated with highest risk of venous thromboembolism (VTE)?
A. Anti-thrombin deficiency.
B. Protein C deficiency.
C. Protein S deficiency.
D. Factor V Leiden.
E. Lupus anticoagulant.

A

**A. Anti-thrombin deficiency. **

In addition to conditions such as immobility, obesity, oral contraceptive pills, etc., there are a variety of inherited factors that contribute to VTE. These factors are also known as strong, medium and weak risk factors.

Anti-thrombin deficiency, protein C deficiency (option B), and protein S deficiency (option C), are associated with high risk of VTE. Among this group members, anti-thrombin deficiency, formerly known as anti-thrombin III deficiency, confers the highest risk of VTE..

(Option D) Factor 5 Leiden is a moderate risk factor for VTE.

(Option E) Lupus anticoagulant is also a factor for both VTE and arterial thromboembolism but is an acquired condition not an inherited one. The prevalence is 1 to 5% of population and even more in the elderly and those with a comorbid condition such as cancer. The risk of developing VTE in an individual with lupus anticoagulant is approximately 6-8%.

References * American Heart Association - Risk Factors for Venous Thromboembolism * MedScape - Genetics of Venous Thromboembolism * PubMed - Venous Thromboembolism: Classification, Risk Factors, Diagnosis, and Management

21
Q

A 7-year-old girl is brought to your practice by her parents, who are extremely concerned about several bruises on the child’s arms, legs and trunk she has developed over the past 2 days. She usually enjoys good health and has had no recent medical problem except an upper respiratory tract infection (URTI) 2 weeks ago with complete resolution. On examination, there are several non-blanching petechial over the trunk and limbs and some bruises. There has been no bleeding event other than bruises and the petechial rash. The rest of the exam is insignificant with no lymphadenopathy or hepatosplenomegaly. A full blood exam is arranged, the result of which is as follows:
- WBC: 10500 (normal: 4,000-11,000)
- RBC: 5x1012/L (normal: 3.9-5.6x1012/L)
- Hb: 13g/dL (normal : 11.5-16g/dL)
- MCV: 85fL (normal: 76-96fL)
- Platelet count: 35x109/L (normal: 150-400x109/L)

Which one of the following is the most appropriate next step in management of this child?
A. Strict bed rest.
B. Systemic steroids.
C. Intravenous immunoglobulin.
D. Platelet transfusion.
E. No treatment.

A

E. No treatment.

The clinical picture of bruising and petechial rash following a viral illness in an otherwise healthy child (no pallor, no lymphadenopathy, and no splenomegaly) is suggestive of immune thrombocytopenic purpura as the most likely diagnosis. Isolated thrombocytopenia found on FBE makes this diagnosis almost certain.

ITP is acute in approximately 90% of cases and spontaneous resolution often occurs without any active treatment. ITP persisting beyond 6 months is termed as chronic ITP.

Management of acute ITP in children includes no treatment at all or oral steroids. Without treatment, the platelet count will return to an acceptable level at which normal activity can be recommended within 4 to 6 weeks. Addition of systemic steroids raise the platelet count faster compared to no treatment. Use of steroids has not shown any significant benefit in terms of mortality, morbidity and risk of chronicity of ITP. Immunoglobulins are considered if there is significant bleeding. Considering this fact, watchful waiting is the treatment option of choice for acute ITP in children where possible.

In general, conservative management with no active treatment should be considered the first-line treatment option the in children with ITP with a platelet count of >20x109/L and some with a platelet count of <20x109/L provided that the following criteria are met:
- The diagnosis is certain.
- There is no pallor, hepatosplenomegaly, or lymphadenopathy.
- There is only isolated thrombocytopenia on FBE.
- There is no active bleeding other than bruising and petechial in isolation. Any mucosal, gastrointestinal, or genitourinary bleeding, even if ceased, excludes the patient from this approach.
- The child is otherwise well.
- Parents can be reassured and educated adequately and appropriately.
- There is confidence about the degree of parental supervision and safety of home environment, particularly for younger children.
- Follow up within a few days is guaranteed by the on-call pediatrician who must be contacted and agreed with the management plan.

In this child a definitive diagnosis of ITP can be made with high certainty. There is only isolated thrombocytopenia of greater than 20x109/L on FBE. He is otherwise healthy has no bleeding other than bruising and petechiae. For her, no treatment and watchful waiting is the most appropriate option provided than follow up is guaranteed, her carers can be reassured and educated, and there is appropriate home safety for the child.

(Option A) While limited activity as means of decreasing a bleeding event is recommended, strict bed rest is not an appropriate advice.

(Option B) Systemic corticosteroids are first-line treatment in children who have active bleeding other than bruises and petechiae, even if ceased now. Prednisolone (2-4mg/kg/day for 2 weeks then tapered) is the regimen of choice. This child has no active bleeding and a platelet count of >20x109/L. For now, he can be managed conservatively with no active treatment.

(Option C) Intravenous immunoglobulin (IVIG) is the second-line treatment option. An important indication is severe active bleeding unresponsive to steroids.

(Option D) In ITP, platelets are sequestrated by the spleen. Platelet transfusion is only rarely considered in cases with severe life-threatening bleeding as a temporary measure, or after splenectomy due to severe persistent ITP where the risk of intra- or post-operative bleeding is significant.

References * http://www.rch.org.au/clinicalguide/guideline_index/Immune_Thrombocytopenic_Purpura/ * https://emedicine.medscape.com/article/202158-treatment

22
Q

A 4-year-old boy is brought to the emergency department by his parents with bruising and a generalized rash for the past two days. He also had an episode of nose bleed this morning that stopped after 10 minutes of pressure and ice application. He also had a runny nose, mild fever and sneezing 10 days ago that was resolved after one week of conservative management. On examination, the child is otherwise healthy and in good shape. His vital signs are stable. There are three bruises over the abdomen, right arm and left buttock as well as generalized non-blanching petechial rash over the trunk and limbs. There is no pallor, lymphadenopathy, or hepatosplenomegaly. A full blood count is arranged with the following results:

  • WBC: 9700 (normal: 4,000-11,000)
  • RBC: 4.2x1012/L (normal: 3.9-5.6x1012/L)
  • Hb: 13.7g/dL (normal : 13-18g/dL)
  • MCV: 90fL (normal: 76-96fL)
  • Platelet count: 56x109/L (normal: 150-400x109/L)

Which one of the following is the most appropriate next step in management?
A. Oral prednisolone.
B. Intravenous immunoglobulin.
C. Platelet transfusion.
D. Strict bed rest.
E. No treatment.

A

**A. Oral prednisolone. **

Petechial rash, easy bruising and isolated thrombocytopenia on FBE in an otherwise healthy child without hepatosplenomegaly, lymphadenopathy or other alarming findings is suggestive of immune thrombocytopenic purpura as the most likely diagnosis. The presence of a recent viral illness in the history makes such diagnosis even more certain

Management of acute ITP in children includes no treatment at all or oral steroids. Without treatment, the platelet count will return to an acceptable level at which normal activity can be recommended within 4 to 6 weeks. Addition of systemic steroids raise the platelet count faster compared to no treatment. Use of steroids has not shown any significant benefit in terms of mortality, morbidity and risk of chronicity of ITP.

In general, conservative management with no active treatment should be considered the first-line treatment option the in children with ITP with a platelet count of >20x109/L and some with a platelet count of <20x109/L provided that the following criteria are met:
- The diagnosis is certain.
- There is no pallor, hepatosplenomegaly, or lymphadenopathy.
- There is only isolated thrombocytopenia on FBE.
- There is no active bleeding other than bruising and petechial in isolation. *Any mucosal, gastrointestinal, or genitourinary bleeding, even if ceased, excludes the patient from this approach. *
- The child is otherwise well. Parents can be reassured and educated adequately and appropriately.
- There is confidence about the degree of parental supervision and safety of home environment, particularly for younger children.
- Follow up within a few days is guaranteed by the on-call pediatrician who must be contacted and agreed with the management plan.

In this child a definitive diagnosis of ITP can be made with high certainty. There is only isolated thrombocytopenia of greater than 20x109/L on FBE. He is otherwise healthy but has had an episode of nose bleed. For this child, oral steroids are indicated because he does not fulfill the criteria for “no treatment” option. Prednisolone (2-4mg/kg/day for 2 weeks then tapered) is the regimen of choice.

(Option B) Intravenous immunoglobulin (IVIG) is the second-line treatment option. An important indication is severe active bleeding unresponsive to steroids.

(Option C) In ITP, platelets are sequestrated by the spleen. Platelet transfusion in limited cases with severe life-threatening bleeding as a temporary measure, or after splenectomy due to severe persistent ITP where the risk of intra- or post-operative bleeding is significant.

(Option D) While limited activity as means of decreasing a bleeding event is recommended, strict bed rest is not an appropriate advice.

(Option E) ‘No treatment’ could be an acceptable option if there was no bleeding in history other than the bruises and petechial rash. With the nose bleed, even though it was minimal and self-limited, systemic steroids should be started as there is the risk of further bleeding episodes that could be significant or even life-threatening.

References * http://www.rch.org.au/clinicalguide/guideline_index/Immune_Thrombocytopenic_Purpura/ * https://emedicine.medscape.com/article/202158-treatment

23
Q

After a long flight from London to Sydney, a 35-year-old man, who is a known case of sickle cell disease, presents to the emergency department with severe right calf pain. The pain has started acutely two hours ago and is 6 out of 10 on a scale of 1 to 10, according to the patient. He denies any chest pain or shortness of breath. On examination, he has a blood pressure of 134/78 mmHg, pulse rate of 98 bpm, respiratory rate of 17 breaths/min, a temperature of 37.2℃, and O2 saturation of 94% on room air. Chest examination is normal and there is no abnormal respiratory or heart sound. Which one of the following would be the next best step in management?
A. Enoxaparin.
B. Adequate analgesia.
C. Oxygen.
D. Hydration.
E. Limb elevation.

A

**B. Adequate analgesia. **

This is a rather challenging question, describing a person with sickle cell disease, who has developed calf pain after a long flight. Long distance flights are associated with prolonged immobility and, consequently, venous stasis that predisposes to deep vein thrombosis (DVT). On the other hand, especially during unpressurized long flights, hypoxia can develop and trigger a sickle cell crisis. There is no additional information such as leg circumference discrepancy, warmth, erythema, etc., as pointers towards DVT versus sickle cell crisis.

DVT can present with local tenderness, erythema, pain, and increased limb circumference. Pain and tenderness is also a feature in patients with pain crisis due to sickle cell disease. However, the most important notion to consider in this scenario is not a clear diagnosis. It is about the general rules to consider in approaching every single patient.

Evaluation of a patient always starts with checking if he/she is stable or not. A stable patient has a patent airway, breathes normally and has normal circulatory parameters such as blood pressure and pulse rate in adults and pulse rate and capillary refill time (CRT) in children. Chest pain and altered consciousness are other indicators of instability to look for. This patient is speaking and giving history. These indicate a patent airway. He is breathing normally and has an acceptable O2 saturation. Therefore, oxygen (option C) is not necessary for now. His pulse rate and blood pressure also provide assurance as to his stable circulatory status. He does not have chest pain and is conscious and oriented. This patient is stable for now. After stability is assured, the next step in priority should be removing any discomfort or pain if possible and putting the patient at ease. In practice and when pain is a symptom, the patient should be asked about the pain characteristics, and if severe and discomforting for the patient, analgesia have to be offered. So, for this patient, whose main complaint is pain, the most appropriate next step in management after the ABC is analgesia regardless of the etiology. Analgesia is also the mainstay of treatment if the diagnosis turns out to be a pain crisis due to sickle cell disease.

Other options should be considered next based on the diagnosis. Enoxaparin (option A) and limb elevation (option E) may be considered if further assessment establishes DVT as the diagnosis. It is important to note that anticoagulation only starts if the a diagnosis of DVT is certain via pretest probability tools and appropriate imaging studies. Hydration (option D) to maintain the plasma volume may be considered if the patient is found to have a pain crisis due to sickle cell disease.

References * NCBI – Sickle Cell Disease and Venous Thromboembolism * Merck Manual – Sickle Cell Disease * RCH – Sickle cell disease * Medscape - Sickle Cell Anemia

24
Q

Peter, 25 years of age, presents to the Emergency Department with generalized severe pain in his limbs and joint. He is a known case of sickle cell disease and has just arrived from Paris to Melbourne after a long flight. On scale of 1 to 10, peter rates the pain as 7. He denies any breathlessness or chest pain. On examination, he has blood pressure of 140/87 mmHg, pulse rate of 104 bpm, respiratory rate of 18 breaths per minute and a temperature of 36.9℃. The only remarkable exam finding is the generalized tenderness on palpation of his muscles, bones, and joints of upper and lower limbs. Which one of the following would be the next best step in management?
A. Oxygen.
B. Hydration.
C. Enoxaparin.
D. Analgesics.
E. CTPA.

A

**D. Analgesics. **

The scenario describes a case of sickle cell crisis, presented with an acute pain crisis. Pain (vaso-occlusive) crises are the most common presentation of sickle cell crises. A painful crisis is caused by ischemia and infarction that typically occurs in the bones, but can also affect the spleen, lungs, or kidneys. With painful crisis, analgesics should be started promptly as the most important initial step. In mild cases, simple analgesics such as paracetamol plus codeine adequately control the pain; however, most patient have more severe pain and often will need intravenous opioids. IV morphine (continuous or bolus) is effective and safe. Meperidine should be avoided.

Although dehydration contributes to sickling and may precipitate crises, it is unclear whether vigorous hydration (option B) is helpful during crises. Nevertheless, maintaining normal intravascular volume if the patient is volume depleted is an important part of management. Patients may even need transfusions using WBC filtered blood.

Although Peter has had a recent long flight, he does not have symptoms suggestive of pulmonary embolism such as chest pain or shortness of breath. Therefore, CTPA (option E) or enoxaparin (option C) are not correct options for now.

Supplementation with oxygen (option A) is required when there is hypoxia, or there is chest syndrome as a manifestation of his sickle crisis. Peter is not hypoxic, nor does he have chest pain; therefore, he is not in immediate need for oxygen.

NOTE – Regardless of the cause, checking for airway, breathing and circulation (ABC) followed by assessment of pain and pain control if necessary is the common approach for every single patient presenting with a medical problem.

Summary of SCD topic review:
Complication of SCD: sickle cell crisis
Can be precipitated by
- Infections
- Dehydration
- Hypoxia
- Sedatives and local anesthesia
Acute crisis can be presented as (basically think of vaso-occlusion in different parts of body):
1. pain crisis (most common) d/t infarction of the bone, spleen, lungs or kidneys- mx: analgesia (analgesics choice depends on PS, according to pain ladder), can give abx if +fever
2. Acute chest syndrome- sx: chest pain, hypoxia; results from pulmonary microvascular occlusion; common COD
3. Aplastic crisis
4. Priapism- painful erection of penis
5. acute neurological events
6. acute splenic sequestration- acutely enlarged spleen
7. Others: leg ulcers, avascular necrosis, and renal involvement.

Detailed Topic review on sickle cell disease in 628-629

References * NCBI – Sickle Cell Disease and Venous Thromboembolism * Merck Manual – Sickle Cell Disease * RCH – Sickle cell disease * Medscape - Sickle Cell Anemia

25
Q

After flying from Sydney to Port Vila, a 25-year-old man develops shortness of breath shortly after landing. On further inquiry, you realize that he is a known case of sickle cell trait, whose father died of myocardial infarction at the age of 36 years. He denies any chest pain and muscle and joint pain but feels slightly short of breath. On examination, he has central cyanosis, a blood pressure of 130/85 mmHg, a pulse of 110 bpm and a respiratory rate of 24 breaths per minute. He is afebrile. Pulseoxymetry shows an O2 saturation of 86% while on room air. The rest of the exam including a full neurological exam is inconclusive. Which one of the following is the next best step in management?
A. Start intravenous heparin immediately.
B. Do a chest X-ray to confirm pulmonary embolism.
C. Do an ECG.
D. Do CTPA to confirm pulmonary embolism.
E. Give oxygen and review.

A

E. Give oxygen and review.

Patients who are heterozygous (Hb AS) (Sickle cell trait) do not experience hemolysis, painful crises, or thrombotic complications except possibly during hypoxic conditions (e.g. at high altitudes, during sudden decompression in airplanes). In these patients, rhabdomyolysis and sudden death may occur during sustained, exhausting exercise. Impaired ability to concentrate urine (hyposthenuria) is common. Unilateral hematuria (by unknown mechanisms and usually from the left kidney) can occur but is self-limited. Typical renal papillary necrosis can occur but is less common than in homozygous patients.

Main presenting symptoms in this man are shortness of breath and central cyanosis. There are conditions to consider as the cause of such clinical picture i.e., long flight in high altitude, pulmonary embolism, acute chest syndrome in the setting of sickle cell trait triggered by hypoxia, etc. However, the most important step to take first is attending to his hypoxia. This patient should be started immediately on oxygen and his status reviewed. With a desirable response that sustains after discontinuation of oxygen, other causes such as acute chest syndrome or pulmonary embolism are unlikely. With sustained hypoxia despite initial oxygen supplementation, other measures to consider are a chest X-ray (option C) to exclude other conditions, an ECG (option C), and CTPA (option D) if there is a high probability for pulmonary embolism. Heparin (option A) is only used when there is a definite diagnosis of pulmonary embolism. Heparin is never given empirically for treatment of venous thromboembolism.

TOPIC REVIEW
Common causes of central cyanosis:
- Upper respiratory tract obstruction
- Impaired gas exchange secondary to pneumonia
- Pulmonary embolism and ventilation/perfusion mismatch
- Impaired gas diffusion in the alveoli
- High altitude
- Anatomic shunts
- Right to left shunts in congenital heart disease
- Arteriovenous malformation
- Intrapulmonary shunt

References * NCBI – Sickle Cell Disease and Venous Thromboembolism * Merck Manual – Sickle Cell Disease * RCH – Sickle cell disease * Medscape - Sickle Cell Anemia

26
Q

A 3-year-old boy is being evaluated in your general practice. For the past 2 days, he has had bruising over his trunk and limbs and a petechial rash over his body. This morning he had an episode of nose bleed lasting about 10 minutes. On examination, he looks well and healthy with no distress. He has no fever. There is no splenomegaly or hepatomegaly. He has several non-blanching petechiae over his trunk and legs and recent bruises on his body. Which one of the following is the most likely diagnosis?
A. Von Willebrand disease.
B. Henӧch-Schonlein purpura.
C. Acute lymphocytic leukemia.
D. Immune thrombocytopenia.
E. Hemophilia A.

A

**D. Immune thrombocytopenia. **

Petechiae indicate that the problem is either a vascular or platelet problem. Petechiae do not occur in factor types of bleeding (e.g. hemophilia or Von Willebrand disease). The clinical picture is highly-suggestive of immune thrombocytopenic purpura, characterized by recent-onset easy bruising, bleeding and petechiae in an otherwise healthy child. The history of a preceding upper respiratory tract infection is another pointer towards the diagnosis.

ITP is an acquired immune-mediated thrombocytopenia characterized by shortened lifespan of circulating platelets in the absence of other disturbances of hemostasis or coagulation.

Patients fall broadly into two categories:
- Acute (~ 90%): self limiting disease (sometimes preceded by a viral syndrome) with spontaneous resolution within 6 months (usually within 2 months)
- Chronic (~ 10%): persists beyond 6 months.

In acute ITP, the patient often presents with bruising and petechiae alone; however, in some instances there is oral bleeding, epistaxis, rectal bleeding or hematuria.

Morbidity in ITP is usually minimal and parents need to be reassured. The incidence of intracranial hemorrhage is less than 1%, and very rare in true uncomplicated ITP.

The clinical diagnosis of ITP relies on manifestations of thrombocytopenia (platelet type of bleeding) without other abnormal findings, in particular no pallor, lymphadenopathy or hepatosplenomegaly. ITP is a diagnosis of exclusion. Other causes of thrombocytopenia should be ruled out.

(Option A) Von Willebrand disease is a hereditary bleeding disorder presenting similar to ITP with easy bruising, petechiae, epistaxis, and menorrhagea in women. The disorder often runs in families. Patients may become symptomatic at any age. Although the presentation of von Willebrand disease shares many features with ITP, two points make it less likely than ITP. First is the fact that petechiae are a less common finding in von Willebrand, and second is the absence of a family history.

(Option B) Henӧch-schonlein purpura is usually associated with abdominal pain and arthralgia which is not present in this child.

(Option C) In acute lymphocytic leukemia, the clinical picture is more pronounced and the child has been unwell for several weeks prior to coagulopathy.

(Option E) Hemophilia is a hereditary condition presenting early in life with factor type of bleeding. Petechiae is not a feature.

References * RCH - Immune thrombocytopenic purpura

27
Q

A 22-year-old man presents with complaints of weakness and lethargy for the past few months. He denies any abdominal pain, rectal bleeding, or bowel symptoms. His past medical history is not remarkable. A full blood exam (FBE) and iron studies are ordered with the following results:
- RBC: 3.5x1012/L (4.5-6.5x1012/L)
- WBC: 6.3x109/L (4.0-11x109/L)
- PLT: 210x109/L (150-400x109/L)
- Hemoglobin: 70 g/L (130-180g/L)
- MCV: 66 fL (76-96 fL)
- Ferritin: 15μg/L (30-200μg/L)
- Serum iron: 7µmol/L (10.74 – 30.43 µmol/L) Which one of the following options is most likely to help reach a diagnosis
A. Bone marrow biopsy.
B. Small bowel biopsy.
C. Hemoglobin electrophoresis.
D. Colonoscopy.
E. Sigmoidoscopy.

A

**D. Colonoscopy. **

According to WHO, anemia is defined as hemoglobin (Hb) <120 g/L for females and <130 g/L for men. According to Royal College of Pathologists of Australasia iron deficiency is present when serum ferritin level is less than 30 µg/L for an adult. Therefore, this patient has iron deficiency anemia. Iron deficiency is never a final diagnosis in itself and a cause should always be sought.

The most common cause of iron deficiency is ongoing bleeding, either overt (e.g., heavy periods, or occult such as that occurring in gastrointestinal bleeding or malignancies). In cases with confirmed iron deficiency anemia for which an obvious source of blood loss cannot be identified in history or physical examination, upper and lower gastrointestinal investigations should be considered. In fact, upper and lower gastrointestinal tract scope are mandatory in all postmenopausal females and all males with confirmed iron deficiency anemia unless there is a history of significant overt non-GI blood loss such as vaginal bleeding.

In premenopausal women with iron deficiency anemia, upper and lower gastrointestinal (GI) investigation should be reserved for those aged over 50 years who have symptoms suggestive of GI disease or a strong family history of colorectal cancer.

This patient is a male with iron deficiency anemia. Since there is not apparent cause for the blood loss, and since the GI tract is the most common site of occult blood loss in such patients, the first and most important step to consider is GI investigations starting with a colonoscopy. If no identifiable is found in colonoscopy, upper endoscopy should follow. Sigmoidoscopy (option E) is not an appropriate choice for this purpose because it only provides information about the sigmoid colon and will miss more proximal potential sources of bleeding.

Celiac disease should also be considered as a differential diagnosis and excluded in all patients with iron deficiency anemia using serologic studies. These tests are screening tests and those with positive test results will require small bowel biopsy (option B) as the confirmatory test. Not every patient with iron deficiency anemia undergoes small bowel biopsy.

(Option A) Bone marrow biopsy and aspiration may be used for many conditions, such as leukopenia, leukocytosis, thrombocytopenia, thrombocytosis, pancytopenia and polycythemia. This test would be used if anemia could not be justified by iron deficiency, or if the patient, besides the anemia, has any of the above features.

(Option C) Serum electrophoresis is the test of choice to confirm thalassemia as the cause of anemia. Although the full blood exam (FBE) of this patient resembles thalassemia trait, the low ferritin level in this patient is against it because in patients with thalassemia, ferritin and iron or either normal or elevated. Since thalassemia cannot cause iron deficiency anemia, serum electrophoresis adds no diagnostic value in this patient.

References * MJA - Diagnosis and management of iron deficiency anaemia: a clinical update

28
Q

A 70-year-old woman presents to your practice with complaints of weakness and easy fatigability for the past 5 months. She is a recent immigrant from Algeria landing on Australia almost 8 months ago. She has been a vegetarian most of her adult life. Past medical history is insignificant for any chronic condition otherwise. Based on the history and physical findings, you suspect anemia and order a full blood examination (FBE) which is significant for a red cell count (RCC) of 2.9x1012/L (4.5- 6.5x1012/L), hemoglobin of 90 g/L (120-160 g/L), MCV of 62 fL (80-100 fL). Which one of the following could be the most likely cause of this presentation?
A. Cecal cancer.
B. Thalassemia minor.
C. Hookworm infestation.
D. The vegetarian diet.
E. Vitamin B12 deficiency.

A

**C. Hookworm infestation. **

The decreased RCC and hemoglobin in this patient represent anemia. With the decreased MCV, the anemia is microcytic. The most common causes of microcytic anemia are iron deficiency and thalassemia. This clinical picture, however, is inconsistent with thalassemia minor (trait) (option B) because although patients with thalassemia minor has isolated mild microcytic anemia, the red cell count, unlike in this patient, is normal of even increased. Moreover, the anemia of thalassemia minor is asymptomatic.

Strict vegetarians are prone to vitamin B12 deficiency as this vitamin is only available in animal products such as meat, poultry, fish, eggs, and dairy products. Vitamin B12 deficiency (option E) causes macrocytic anemia that does not explain the microcytosis in this scenario.

A vegetarian diet includes green-leaf vegetables that provide adequate dietary iron to prevent from iron deficiency. Anemia caused by vegetarian diet (option D) is more likely to be macrocytic and have been caused by vitamin B12 deficiency.

Cecal cancer (option A) and right-sided colon cancer in general often present with anemia and its symptoms such as fatigue and weakness (as opposed to left-sided colorectal cancers that usually manifest with altered bowel habits and rectal bleeding); therefore, should be considered and investigated in patients with microcytic anemia, or iron deficiency anemia to be more specific. This is even more important in older patients. However, colorectal cancer in vegetarians is extremely rare because high-fiber diets are a protective factor against colorectal cancer.

Given the history and exclusion of other options, hookworm infestation remains the most likely explanation to this scenario.

Human hookworm disease is a common helminth infection predominantly caused by the nematode parasites Necator americanus and Ancylostoma duodenale. Hookworm infection is acquired through skin exposure to larvae in soil contaminated by human feces.

Worldwide, hookworms infect an estimated 472 million people. It is endemic in many underdeveloped or developing regions in the work including African, south Asia, Mediterranean region, and south America.

The larvae of hookworm from the soil enter the foot sole skin, then migrate through the dermis, enter the bloodstream, and move to the lungs within 10 days. Once in the lungs, they break into alveoli, causing a mild and usually asymptomatic alveolitis with eosinophilia. Form the alveoli, the larvae are carried to the glottis by the ciliary action of the respiratory tract. During pulmonary migration, the host may develop a mild reactive cough, sore throat, and fever that resolve after the worm migrates into the intestines. At the glottis, the larvae are swallowed and carried to their final destination, the small intestine. Of all infested people, only 10% develop symptoms which include iron deficiency anemia due to loss from small intestine in patients with moderate to severe infestation burden.

References * UpToDate – Hookworm infection

29
Q

A 34-year-old pregnant woman presents to your office for evaluation at 12 weeks gestation. Her obstetrical history is significant for 2 miscarriages in previous pregnancies, for which she underwent assessment and was found to have antiphospholipid syndrome. She is currently on aspirin. Which one of the following would be the next best step in management?
A. Do nothing, as aspirin alone is adequate.
B. Start her on prophylactic dose of low molecular weight heparin.
C. Start her on warfarin.
D. Start her on therapeutic dose of low molecular weight heparin
E. Arrange for elective abortion.

A

**B. Start her on prophylactic dose of low molecular weight heparin. **

For women with laboratory-proven criteria for antiphospholipid antibody syndrome and one or more miscarriages after 10 weeks gestation, combined therapy with low-dose aspirin (50 to 100 mg per day) and prophylactic dose of low molecular weight heparin (LMWH) upon confirmation of intrauterine pregnancy is recommended. Therefore, addition of low-molecule weight heparin (LMWH) in prophylactic dose would be the next best step in management. Treatment should be continued throughout the pregnancy. Therapy is withheld at the time of delivery and is restarted after delivery, continuing for 6-12 weeks postpartum.

References * http://emedicine.medscape.com/article/333221-treat * http://www.uptodate.com/contents/pregnancy-in-wome * http://www.guideline.gov/content.aspx?id=25312

30
Q

A 69-year-old woman has been in the Intensive Care Unit (ICU) for the past 7 days following a complicated hip replacement surgery. She is currently on low-molecular weight heparin (LMWH) as well as intermittent pneumatic compression device. Today, she has developed new-onset right calf pain. On examination, the leg is edematous and tender to touch. A positive Homan’s sign is elicited. A Doppler ultrasound confirms the presence of a clot in the right lower leg and the diagnosis of deep vein thrombosis. Blood tests show a platelet count of 78,000 /m3m, but there is no sign of hemorrhage. Which one of the following is the next best step in management?
A. Discontinue application of pneumatic compression device.
B. Start her on warfarin.
C. Discontinue heparin.
D. Perform venography.
E. Transfuse platelets.

A

**C. Discontinue heparin. **

Development of DVT in the presence of thrombocytopenia is suggestive of heparin-induced thrombocytopenia (HIT) as the most likely diagnosis. HIT is a complication of heparin therapy.

‘There are two types of HIT:’
1. Type 1 HIT - presents within the first 2 days after exposure to heparin, and the platelet count normalizes with continued heparin therapy. Type 1 HIT has a non-immune pathophysiology.
2. Type 2 HIT - an immune-mediated disorder that typically occurs 4-10 days after exposure to heparin and has life- and limb-threatening thrombotic complications. In general medical practice, the term HIT refers to type 2 HIT.

HIT is caused by antibodies that bind to complexes of heparin and platelet factor 4 (PF4), activation of the platelets, and promoting a prothrombotic state. HIT is more frequently encountered with unfractionated heparin (UFH) than with low- molecular weight heparin (LMWH).

The risk of HIT is highest with prolonged use of heparin for postoperative thromboprophylaxis. However, case studies have also shown that HIT can occur even with minimal heparin exposure via intravascular flushes to maintain an arterial or venous catheter open.

HIT must be suspected when a patient, who is receiving heparin, has a decrease in the platelet count, particularly if the fall is over 50% of the baseline count, even if the platelet count remains above 150,000/mm3. Clinically, HIT may manifest as skin lesions at heparin injection sites or by acute systemic reactions (e.g., chills, fever, dyspnea, chest pain) after administration of an intravenous bolus of heparin.

Unlike other forms of thrombocytopenia, HIT is associated with thromboembolism (e.g., DVT, PE) as the most common complication, not with bleeding. Sometimes, arterial thrombosis (e.g., myocardial infarction) may occur. For this reason, the disorder is sometimes termed heparin-induced thrombocytopenia and thrombosis (HITT).

Factors that increase the risk for heparin-induced thrombocytopenia (HIT) are as follows: - Use of unfractionated heparin (UFH) rather than low molecular weight heparin (LMWH)
- Intravenous rather than subcutaneous administration of heparin
- Longer duration of heparin use
- Surgical (especially cardiac or orthopedic surgery) rather than medical patient
- Female sex

HIT usually occurs 4-10 days after the start of heparin therapy. However, in patients with recent prior heparin exposure (especially within the previous month, but even within the past 100 days), persistence of circulating HIT antibodies may result in rapid-onset HIT, in which the platelet count falls within 24 hours of starting heparin. Onset of HIT may also occur after heparin cessation. Delayed-onset HIT should be considered when a patient presents with thrombosis and unexplained thrombocytopenia up to 3 weeks after heparin therapy has ended.

Patients with heparin-induced thrombocytopenia (HIT) may present with unusual characteristic sequelae, including the following:
- Venous limb gangrene
- Bilateral adrenal hemorrhagic infarction
- Skin lesions at injection sites
- Acute systemic reactions following an intravenous heparin bolus

If heparin-induced HIT is suspected, the first step is to discontinue and avoid all heparin products immediately. Even small doses for catheter flushing should be avoided.

(Option A) HIT is not caused by pneumatic compression; therefore, its discontinuation has no benefit in management of this patient.

(Option B) Warfarin should be avoided in such patients because it can lead to microthrombosis. Interestingly, patients with HIT have an INR of above 4. Warfarin can be started once the platelet count is back to normal. If warfarin has already been started, vitamin K should be given.

(Option D) Once a Doppler ultrasound has confirms the diagnosis of DVT, venography is not required as an extra measure unless some surgical intervention is planned and an accurate venous map is required.

(Option E) Platelet transfusion should be avoided in HIT, as it may increase the thrombogenic effect. Guidelines from the American College of Chest Physicians suggest limiting platelet transfusions to patients with severe thrombocytopenia, who are experiencing bleeding or undergoing an invasive procedure with a high risk of bleeding.

Once HIT occurs, alternatives to heparin are direct thrombin inhibitors (DTIs) such as argatroban and the indirect factor Xa inhibitor fondaparinux (Arixtra®).

References * Medscape - Heparin-Induced Thrombocytopenia

31
Q

Which of the following is the most profound adverse effect of chronic transfusional iron overload in children with thalassemia?
A. Progressive hepatic cirrhosis.
B. Extramedullary hematopoeisis.
C. Productive cough and shortness of breath. D. Hemolytic anemia.
E. Autosplenectomy.

A

**A. Progressive hepatic cirrhosis. **

The following are are major complications associated with iron overload:
1. Cardiac problems: Congestive heart failure, dysrhythmias and pericarditis
2. Endocrine problems: Delay in growth and sexual development, hypoparathyroidism, hypothyroidism and diabetes mellitus
3. Hepatic problems: Progressive hepatic fibrosis and cirrhosis Of the given options, only progressive hepatic cirrhosis could be a complication of iron overload.

Of the options, only progressvie hepatic cirrhosis can be direct complication of iron overload.

(Option B) Extra-medullary hematopoiesis is a feature of thalassemia, not a complication of iron overload.

(Option C) Shortness of breath can be caused only if iron overload has already led to heart failure. It is not a direct effect of iron overload.

(Option D) Hemolysis and anemia is the disease mechanism, not an adverse effect of iron overload.

(Option E) Autosplenectomy is a feature of sickle cell disease and occurs because of splenic infarcts.

NOTE - Splenomegaly is not a common feature in thalassemia. If present, it is more likely to have been caused by congestive heart failure rather than the disease itself

References * MJA - Management and clinical outcomes of transfusion-dependent thalassaemia major in an Australian tertiary referral clinic * Australian Guidelines for the assessment of iron overload and iron chelation in transfusion-dependent thalassaemia major, sickle cell disease and other congenital anaemias. *

32
Q

A 70-year-old woman presents with abdominal discomfort of 6 months duration. On examination, generalized lymphadenopathy and splenomegaly are noted. Which one of the following could be the most likely diagnosis?
A. Acute lymphoblastic leukemia (ALL).
B. Acute myeloid leukemia (AML).
C. Hodgkin lymphoma.
D. Infectious mononucleosis.
E. Chronic lymphoblastic leukaemia (CLL).

A

E. Chronic lymphoblastic leukaemia (CLL).

The clinical picture is suggestive of CLL as the most likely diagnosis. CLL is massive overproduction of mature yet till leukemic lymphocytes usually from the monoclonal production of B lymphocytes

CLL can often present as an asymptomatic elevation of white cells, found on routine blood tests or during investigations for other problems. The patients are exclusively older with 90% being over the age of 50 years. Symptoms, when present, are often nonspecific.

CLL can be associated with various autoimmune phenomena such as thrombocytopenia and autoimmune hemolytic anemia

Enlarged lymph nodes are the most common presenting symptom (87% of symptomatic patients) at the time of diagnosis. A predisposition to repeated infections such as pneumonia, herpes simplex labialis, and herpes zoster may be noted. Early satiety and/or abdominal discomfort may be related to an enlarged spleen. Splenomegaly and hepatomegaly are seen in 30-45% and 10-20% of cases, respectively. Thrombocytopenia can cause mucocutaneous bleeding and/or petechiae. Ten percent of patients with CLL will present with an autoimmune hemolytic anemia.

Richter syndrome or Richter transformation refers to the transformation of CLL into an aggressive large B-cell lymphoma and is seen in approximately 3-10% of cases. Patients will often present with symptoms of weight loss, fevers, night sweats, muscle wasting, (i.e. B symptoms) and increasing hepatosplenomegaly and lymphadenopathy. Treatment remains challenging and prognosis poor, with median survival in months.

Staging of CLL is as follows:
- Stage 0 - lymphocytosis alone (usually between 30,000 – 50,000, but may go as high as 150,000 – there is always lymphocytosis)
- Stage 1 - lymphadenopathy
- Stage 2 - splenomegaly
- Stage 3 - anaemia (Hb <100 g/L)
- Stage 4 - thrombocytopenia (<100,000/mm3)

Patients with stages 0 and 1 often have a survival of 10-12 years even without treatment. In contrast, the survival of stage 3 and 4 is only 1-2 years.

Early stage CLL with only an elevated white cell count or enlargement of lymph nodes is not treated; however, patients with symptomatic disease always require treatment. Those with more advanced disease should receive initial therapy with fludarabin. Chlorambucil may be used as second-line therapy. Chlorambucil decreases the white cell count and symptoms.

Prednisolone is used for treatment of thrombocytopenia and autoimmune hemolytic anemia.

(Options A and B) AML and ALL have rapid progressive courses, and without treatment, the survival rate hardly exceeds 12 weeks. ALL is more commonly seen in children. Presentation is with fatigue, anemia, infections and bleeding caused by reduction in all 3 cell lines - red blood cells, white blood cells and platelets.

(Option C) Hodgkin disease presents with enlarged, painless, rubbery, non-erythematous, non-tender lymph nodes as the hallmark of the disease. In Hodgkin disease lymphadenopathy is limited to cervical and supraclavicular lymph nodes 80-90% of the time (unlike non-Hodgkin disease where lymphadenopathy is generalized 80-90% of the time). Drenching night sweats, weight loss of >10% and fever, collectively termed as ‘B symptoms’ are another clinical feature. Pruritus is common. Although extralymphatic involvement (spleen, liver, bone marrow) can be seen, it is more common with non-Hodgkin lymphoma. The disease has a bimodal age distribution at 20s and 60s.

(Option E) Infectious mononucleosis cannot be the diagnosis because firstly, there is no history of fever or pharyngeal symptoms in the history and secondly, acute phase of infectious mononucleosis (lymphadenopathy, fever, pharyngitis) resolve within 1-2 weeks; the fatigue however may last for months. With this presentation, infectious mononucleosis is a very unlikely diagnosis.

References * http://emedicine.medscape.com/article/199313-overv * http://emedicine.medscape.com/article/197802-overv * http://emedicine.medscape.com/article/201886-overv

33
Q

An 83-year-old woman is brought to your attention with complaints of progressive fatigue. On examination, axillary rubbery and non-tender lymphadenopathy is noted. Blood tests show a platelet count of 70,000/mm3, hemoglobin of 70 g/L and WBC count of 40,000/mm3. Which one of the following would be the most appropriate treatment option for this patient?
A. Palliative care.
B. Prednisolone.
C. Platelet transfusion.
D. Radiotherapy.
E. Chemotherapy.

A

**B. Prednisolone. **

Considering the age of the patient and clinical and laboratory findings, chronic lymphoblastic leukemia (CLL) is the most likely diagnosis.

CLL is massive overproduction of mature, but still leukaemic, lymphocytes usually from the monoclonal production of B lymphocytes

CLL can often present as an asymptomatic elevation of white cells found on routine blood tests or during investigations for other problems. The patients are exclusively older with 90% being over the age of 50 years. Symptoms, when present, are often nonspecific,

CLL can be associated with various autoimmune phenomena such as thrombocytopenia and autoimmune hemolytic anemia

Patients with CLL present with a wide range of symptoms and signs. Onset is insidious, and it is not unusual for CLL to be discovered incidentally after a blood cell count is performed for another reason; 25-50% of patients will be asymptomatic at the time of presentation.

Enlarged lymph nodes are the most common presenting symptom, seen in 87% of symptomatic patients at the time of diagnosis. A predisposition to repeated infections such as pneumonia, herpes simplex labialis, and herpes zoster may be noted. Early satiety and/or abdominal discomfort may be related to an enlarged spleen. Splenomegaly and hepatomegaly are seen in 30-45% and 10-20% of cases, respectively. Mucocutaneous bleeding and/or petechiae may be due to thrombocytopenia. Tiredness and fatigue may be present secondary to anemia; 10% of patients with CLL will present with an autoimmune hemolytic anemia.

Patients with chronic lymphocytic leukemia (chronic lymphoid leukemia, CLL) do not need to be treated with chemotherapy until they become symptomatic or display evidence of rapid progression of disease characterized by the following:
- Weakness, night sweats, weight loss, painful lymphadenopathy, or fever
- Symptomatic anemia and/or thrombocytopenia (stages III or IV)
- Autoimmune hemolytic anemia and/or thrombocytopenia poorly responsive to corticosteroid therapy
- Progressive disease, demonstrated by increasing lymphocytosis with a lymphocyte doubling time less than 6 months, and/or rapidly enlarging lymph nodes, spleen, and liver. In contrast, transient localized lymphadenopathy, occurring in response to localized infections, is not necessarily an indication for treatment
- Repeated episodes of infection; hypogammaglobulinemia without repeated episodes of infection is not a clear indication for therapy.

NOTE - Autoimmune mediated anemia or thrombocytopenia is treated with therapy directed at the autoimmune process before treatment of the underlying CLL is initiated.

When chemotherapy is indicated, one of the following regimens is used:
- Fludarabine, cyclophosphamide, and rituximab (FCR)
- Pentostatin, cyclophosphamide, and rituximab (PCR)
- Fludarabine, cyclophosphamide, and mitoxantrone (FCM)
- Cyclophosphamide, vincristine, and prednisone (CVP)

As mentioned earlier, corticosteroids such as prednisolone are first-line treatment where autoimmune complications of CLL such as autoimmune thrombocytopenia or hemolytic anaemia occurs. This treatment should precede chemotherapy (if indicated) when there is thrombocytopenia and/or anaemia.

References * http://www.uptodate.com/contents/overview-of-the-t * http://emedicine.medscape.com/article/199313-treat * http://www.bcshguidelines.com/documents/Revised_CL

34
Q

A 46-year-old man presents with complaint of neck lump that appears and disappears every few weeks. Apart from the lump, there is no other symptom. On examination, a lump is noted in the right anterior triangle of the neck. The lump is rubbery in consistency and non-tender. The rest of the exam is normal. Which one of the following options could be the most likely diagnosis?
A. Acute lymphoblastic leukemia (ALL).
B. Acute myeloid leukemia (CML).
C. Hodgkin lymphoma (HL).
D. Follicular lymphoma.
E. Chronic lymphoblastic leukaemia (CLL).

A

**D. Follicular lymphoma. **

The waxing and waning nature of the lump, that is very likely to be an enlarged lymph node, is most consistent with follicular lymphoma.

Follicular lymphoma (FL, previously called follicle center lymphoma) is a heterogenous clinicopathologic entity that includes tumors derived from germinal center B cells. FL is the second most common subtype of non-Hodgkin lymphoma (NHL).

Most patients FL present with painless peripheral adenopathy in the cervical, axillary, inguinal, and/or femoral regions. The adenopathy commonly waxes and wanes spontaneously, but does not altogether disappear. While hilar and mediastinal nodes are often involved, large mediastinal masses are rare.

Some patients present with relatively asymptomatic large abdominal masses with or without evidence of gastrointestinal and/or urinary tract obstruction. Staging studies usually demonstrate widely disseminated disease with overt involvement of the following extralymphatic sites:
Spleen (40%)
Liver (50%)
Bone marrow (60-80%)

Despite the presence of widespread disease at diagnosis, most patients are asymptomatic other than for lymph node enlargement. Only approximately 20% presents with B symptoms (i.e. fevers, night sweats, or unintentional weight loss). There are no characteristic laboratory abnormalities specifically associated with FL and, despite the large tumor burden, fewer than 25% of patients present with an increased serum lactate dehydrogenase (LDH) or cytopenias in the peripheral blood.

35
Q

A 45-year-old woman is evaluated as a part of routine health insurance medical examination. The past medical history is significant for a gastric bypass surgery 12 years ago. Her blood test report is remarkable for hemoglobin of 115 g/L (normal 130-160g/L), an MCV of 110 fL (normal 80-100 fL) and a platelet count of 110000/mm3 (normal 150000-400000/mm3). The reticulocyte count is 0.5% (normal in adults 0.5-1.5%). Which one of the following is the most appropriate next step in management?
A. Colonoscopy.
B. Thyroid stimulation hormone (TSH) level.
C. Serum vitamin B12 level.
D. Iron studies.
E. Bone marrow biopsy.

A

**C. Serum vitamin B12 level. **

The laboratory findings of macrocytic anemia (MCV>100) and thrombocytopenia in association with the history of gastric bypass surgery makes pernicious anemia the most likely diagnosis, and serum vitamin B12 level for confirmation the best next step in management.

Pernicious anemia is a result of significant decrease in intrinsic factor, a glycoprotein secreted by parietal cells in the gastric mucosa, which is necessary for the absorption of vitamin B12 in the ileum. Antibodies against parietal cell in autoimmune diseases or gastric bypass surgery are common causes of pernicious anemia. It presents with macrocytic anemia, low platelet and normal reticulocyte count. A strict vegetarian diet is another cause.

(Option A) Colonoscopy is the best diagnostic tool when approaching to patients with unexplained iron deficiency anemia because GI tract is a main cause of occult bleeding in such patients. This patient, however, has macrocytic anemia. Iron deficiency presents with microcytic anemia.

(Option B) Although hypothyroidism is one of the differential diagnosis of macrocytic anemia, but the history and the low platelet count makes it less likely as the etiology of the anemia.

(Option D) Iron study is not the most appropriate step as the anemia is not typical of iron deficiency which presents with microcytosis.

(Option E) Bone marrow biopsy is indicated where no cause for the anemia can be found or is the cause is suspected to originate from bone marrow problems. It is not indicated here, at least for now.

References * Medscape - Pernicious Anemia * UpToDate - Treatment of vitamin B12 and folate deficiencies

35
Q

Joseph, 5 years old, is brought to the emergency department by his mother. About one hour ago and while running at home, he hits his face against the coffee table and gets a nose bleed. The mother tried to stop the bleeding by compressing the nose and applying ice, to no avail. On physical examination, the child looks otherwise quite healthy with no remarkable finding. A full blood exam (FBE) is inconclusive. Which one of the following could be the most likely cause of this persistent bleeding?
A. Immune thrombocytopenic purpura (ITP).
B. Von Willebrand disease.
C. Hemophilia.
D. Leukemia.
E. Factor V Leiden mutation.

A

**B. Von Willebrand disease. **

The Scenario describes a healthy child with a platelet type of bleeding. In bleeding disorders with faulty platelet or platelet-mediated coagulation, the bleeding tends to occur immediately after insult and is superficial.

In patients with factor deficiency (factor type of bleeding) the bleeding is deep (e.g. into muscles and joints) and delayed, because initially platelets will establish homeostasis.

Von Willebrand disease (VWD) is an inherited disease with many different types (22 types) and clinical pictures. Almost all types cause mild bleeding problems with excellent prognosis.

The most common types often have an autosomal dominant inheritance. This disease is very common (1 in 100 population) and is the most common inherited bleeding disorder. Von Willebrand factor is a circulating factor that is attached to factor VIII. This factor by gluing platelets together and to the vascular lining plays the earliest role in coagulation.

Because the clinical disease can be very mild, most cases will never be diagnosed, but if symptomatic, the symptoms can include:
- Easy bruising
- Mucosal bleeding (e.g. epistaxis, menorrhagea, gastrointestinal bleeding, etc)
- No history of haemarthroses or intramuscular hematomas (except for type 3 which is very rare but can have musculoskeletal manifestations)
- Prolonged bleeding after trauma or surgery

The clinical picture and a normal platelet count suggests VWD as the most likely diagnosis.

(Option A) Immune thrombocytopenic puprpura (ITP), as the name implies, should have low platelet count as a feature, which is not present in this patient.

(Option C) Hemophilia presents with factor type of bleeding (deep and delayed).

(Option D) With isolated thrombocytopenia and an otherwise healthy child leukemia is very unlikely if possible at all.

(Option E) Factor V Leiden mutation presents with hyper-coagulability not bleeding disorders.

35
Q

A 54-year-old man from India presents to your clinic with tiredness and fatigue for the past 12 months. He is a strict vegatarian. He does not consume alcohol or smoke. He denies abdominal symptoms. On examination, he is very pale. The rest of the examination is unremarkable. Blood test show a low hemoglobin (90 g/L) and an MCV of 130 fL (normal 76-96 fL). Which one of the following is the most likely cause of his presentation?
A. Folic acid deficiency.
B. Thalassemia.
C. Vitamin B12 deficiency.
D. Lead poisoning.
E. Iron deficiency.

A

**C. Vitamin B12 deficiency. **

With a hemoglobin of 9 g/L and n MCV of 130 fL, this patient has macrocytic anemia. Macrocytic anemia is most commonly seen in B12 and B9 (folate) deficiency. This patient is a strict vegetarian. Plants are poor in B12; therefore, B12 deficiency and consequent megaloblastic macrocytic anemia best explains his anemia.

Vitamin B12 deficiency can increase serum homocysteine that is an amino acid implicated as a strong risk factor for cardiovascular diseases. Neurological problems are other complications.

Major risk factors for vitamin B12 deficiency include:
- Age more than 65 years
- Gastric bypass surgery or gastric resection (intrinsic factor deficiency)
- Terminal ileum disease (Crohn’s disease)
- Metformin use
- Strict vegetarian diet

Folic acid (option A) deficiency can also cause macrocytic anemia, but vegetarians receive adequate amounts of it through their diet. So, folic acid deficiency is less likely to be the cause of anemia in this man.

Thalassemia (option B), lead poisoning (option D) and iron deficiency (option E) present with microcytic anemia and cannot explain the macrocytosis in this patient.

References * Medscape - Pernicious Anemia * UpToDate - Treatment of vitamin B12 and folate deficiencies

36
Q

A patient is booked for splenectomy due to chronic idiopathic thrombocytopenic purpura. Which one of the following is the most appropriate management regarding platelet transfusion?
A. Give 6 units of platelets and 6 units of FFP the night before the surgery.
B. Give 10 units of platelets the night before surgery.
C. Give 6 units of platelets the night before surgery.
D. Give 6 units of platelets after clamping the splenic artery.
E. Give 6 units of platelets the night and another 6 units in the morning before surgery.

A

**D. Give 6 units of platelets after clamping the splenic artery. **

The problem in ITP is that platelets are cleared from the blood by the spleen; therefore giving platelets while the spleen is in place is futile and is reserved for occasions when bleeding is life-threatening.

On these occasions, platelets are considered as a short-lived management until more effective measures could be taken. As there is no comment regarding the severe bleeding, the platelets should be given to the patient for post-op homeostasis once the splenic artery is clamped and the spleen is no more able to sequestrate platelets.

  • http://emedicine.medscape.com/article/1829892-over
36
Q

Which one of the following is the best test to diagnose Fanconi’s anemia?
A. Bone marrow biopsy.
B. Full blood count.
C. Chromosome fragility test.
D. Genetic testing.
E. Presence of neutropenia.

A

**C. Chromosome fragility test. **

Fanconi anemia is the most frequently reported of the rare inherited bone marrow failure syndromes. There has been only 2000 cases reported in the medical literature. Fanconi anemia is the most common inherited cause of aplastic anemia.

The transmission mode is autosomal recessive. Fanconi anemia is different from Fanconi syndrome which is also a rare hereditary kidney disease.

The pathophysiology of the disease is a faulty mechanism in DNA repair. In Fanconi anemia, chromosomes are susceptible to DNA cross-linking agents. About 75% of patients with Fanconi anemia have birth defects, such as skin problems (e.g. café au lait spots) (>50%), short stature (50%), thumb or thumb and radial anomalies (40%), abnormal male gonads (30%), microcephaly (25%), eye anomalies (20%), structural renal defects (20%), low birth weight (10%), developmental delay (10%), and abnormal ears or hearing (10%).

Once Fanconi anemia is suspected, the patient should be referred to a hematologist and/or geneticist, who can arrange for diagnostic testing. The laboratory should be accredited and certified to perform Fancoi anemia, and should have evaluated many patients with and without FA because evaluating a large number of patients enables a laboratory to validate its Fanconi anemia testing procedures, and to establish ranges for normal and abnormal test results.

The recommended testing procedures includes:
1. Chromosome breakage tests also termed chromosome fragility test
2. Mutation analyses
3. Bone marrow chromosome analyses

Of these three set of tests, a chromosome fragility test is the first and most important test to consider and perform. In this test, DNA cross-linking agents are used on cells cultured from the patient’s blood sample. Normal cells can correct most of the chromosomal damage caused by the DNA cross-linking agents, whereas cells from patients with Fanconi anemia A typically show multiple chromosomal breaks and rearrangements.

  • Fanconi Anemia: Guidelines for Diagnosis and Management * Medscape - Fanconi Anemia
36
Q

A 25-year-old woman presents with a history of recurrent nose bleeds and easy bruising. Investigations confirms the diagnosis of acute idiopathic (immune) thrombocytopenia purpura (ITP). Her hemorrhage tendency is best initially treated with which one of the following?
A. Gamma globulin infusion.
B. Platelet transfusion.
C. Plasma infusion.
D. Corticosteroids.
E. Azathioprine.

A

**D. Corticosteroids. **

Initial therapy for bleeding tendency in ITP in adults is with corticosteroids. Corticosteroids are thought to reduce the clearance of platelets from the circulation.

(Option A) Intravenous gamma globulin infusion is indicated if there is inadequate response to high-dose corticosteroids.

(Option B) Since platelets are rapidly cleared from the blood in ITP, platelet transfusion is of little benefit in long-term management of the disease. However, it may be considered for life-threatening significant bleeding or prior to emergency splenectomy.

((Opion E) Azathioprine is not used as initial therapy, but may be considered as a corticosteroid-sparing agent for long-term management.

(Option C) Plasma infusion has no role in management of ITP.

References * http://www.ebmt.org/Contents/Resources/Library/Res * http://www.rch.org.au/clinicalguide/guideline_inde

36
Q

A 6-year-old girl with sickle cell disease is brought to your practice for routine follow-up. She is completely asymptomatic and has never had a sickle cell crisis. She takes no medications. Physical examination is unremarkable. Which one of the following complications is more likely to occur earliest in this patient?
A. Splenic infarction.
B. Bone infarction.
C. Splenic sequestration.
D. Stroke.
E. Acute coronary ischemia.

A

B. Bone infarction.

Vaso-occlusive phenomena and hemolysis are the clinical hallmarks of sickle cell disease (SCD) that is an inherited disorder due to homozygosity for the abnormal hemoglobin S (HBs). Vaso-occlusion results in recurrent painful episodes and a variety of serious organ system complications that can lead to lifelong disabilities and early death. Clinical signs and symptoms typically develop at an early age.

Bone infarction is the most common earliest presenting symptoms. This often presents as acute pain in the hands and/or feet, and is often the initial symptom occurring in 40% of all patients in general and 50% of children who become symptomatic before the age of two years. The cause is bone marrow infarction. Since pain tends to involve bones with highest bone marrow activity, and because marrow activity changes with age, different bone pain patterns are predictable. During the first 18 months of life, the metatarsal and metacarpals can be involved, presenting as dactylitis or hand-foot syndrome.

An acute episode of pain is the second most common initial presentation and the most common symptom after the age of two years.

Splenic sequestration is the third most common presenting symptom, occurring in 20% of patients overall and one-third of children before the age two years. Over time, vaso-occlusion can occur in virtually every organ system, accounting for the characteristic acute and chronic multisystem failure associated with the disease.

36
Q

A 23-year-old woman presents to your practice with complaint of breathlessness while climbing stairs. On blood tests, she has hemoglobin of 7 g/dL. Which one of the following is the most appropriate next step in management?
A. Transfusion of packed cells.
B. Colonoscopy.
C. Small bowel biopsy.
D. Parenteral iron.
E. Fecal occult blood test (FOBT).

A

A. Transfusion of packed cells.

According to guidelines, transfusion of packed red cells is not indicated for those with Hb >10 g/dL. The lower threshold varies from 6 g/dL to 8 g/dL; however, hemoglobin level alone cannot be used to determine the need for packed cell transfusion.

It is recommended that symptomatic anemia be treated with transfusion of packed cells in all patients with Hb<10 g/dL, regardless of the Hb level, provided that the symptoms are severe enough and are clearly related to the anemia rather than the underlying condition. This is true regardless of the underlying etiology of the anemia.

Colonoscopy (option B), FOBT (option E), or small biopsy (option C) may be indicated as attempts to spot the source or iron loss or deficiency where iron deficiency is the underlying cause of anemia. Before proceeding to such tests, iron deficiency should be diagnosed by iron studies. Iron replacement (most commonly by oral supplementation) is the treatment of established iron deficiency anemia. Parentral iron (option D) may be used in very severe cases of iron deficiency anemia.

NOTE – According to most references, exertion-related symptoms of anemia are not indications for transfusion of packed cells. In this question, however, the only acceptable option will be transfusion of packed cells, as no other option is correct regarding this clinical scenario.

37
Q

A 23-year-old African man presents to your clinic with pallor and dark urine after he took cotrimoxazole for an acute respiratory infection. Blood investigations are significant for hemoglobin of 90 g/L and a reticulocyte count of 5%. Both direct and indirect Coombs tests are negative. Serum electrophoresis shows type A hemoglobin. There is no family history of such presentation. Which one of the following is the most likely diagnosis?
A. Glucose-6-phosphate dehydrogenase deficiency.
B. Autoimmune hemolytic anemia.
C. Sickle cell anemia.
D. Thalassemia.
E. Hereditary spherocytosis.

A

A. Glucose-6-phosphate dehydrogenase deficiency.

Of the options, glucose-6-phosphate (G6PD) deficiency is the most likely diagnosis.

G6PD deficiency is an X-linked enzymatic defect that can cause hemolysis after oxidative stresses such as acute illness or ingestion of specific drugs. The most commonly implicated drugs are sulfa drugs (e.g., cotrimoxazole), primiquine, dapsone, quinidine and nitrofurantoin. The most common type of oxidant stress, however, is infections not drugs.

Type-A G6PD deficiency is the milder form and is more common among black people, whereas type B is mostly seen in patients of north Mediterranean origin. Type B has a more severe presentation.

The usual presentation is similar to all hemolytic anemias i.e. low hemoglobin, high LDH, decreased haptoglobin, elevated bilirubin (mostly indirect) and increased reticulocyte count in the presence of a normal MCV. An elevated reticulocyte count is a criterion for hemolysis but is not specific for it.

(Option B) Negative Coombs test rules out the possibility of autoimmune hemolytic anemia.

Options C and D) With normal hemoglobin (hemoglobin A, which includes two normal α chains and two normal β chains) on serum electrophoresis, sickle cell disease and thalassemia are excluded.

(Option E) With a negative family history of hereditary spherocytosis, this disease is less likely to be the cause of hemolysis in this patient. Another clue against such diagnosis is that hereditary spherocytosis manifestation occur much earlier in life.

38
Q

A 27-year-old man presents with sudden onset of jaundice, pallor, and dark urine three days after taking of primaquine for malaria. On blood tests, hemoglobin is 52 g/L (120-160g/L) and unconjugated bilirubin is elevated. Blood film shows ‘bite cells’. Which one of the following is the most likely diagnosis?
A. Iron deficiency anemia.
B. Glucose-6-phosphate dehydrogenase deficiency.
C. Anemia due to blood loss.
D. Acute hepatitis.
E. Chronic renal failure.

A

**B. Glucose-6-phosphate dehydrogenase deficiency. **

Development of jaundice after use of primaquine is suggestive of glucose-6-phosphate dehydrogenase (G6PD) deficiency.

The typical presentation is sudden onset of jaundice, pallor, and dark urine, with or without abdominal and back pain. This is associated with an abrupt fall in the hemoglobin concentration of 30-40 g/L during which time the peripheral blood smear reveals red cell fragments, microspherocytes, and eccentrocytes or “bite” cells.

Special stains show Heinz bodies, which are collections of denatured globin chains often attached to the red cell membrane.

Episodes of acute hemolytic anemia are triggered by oxidative stress. Infections are the most common triggering factor.

Iron deficiency anemia, anemia due to blood loss and anemia of chronic renal failure are not associated with elevated unconjugated bilirubin. This fining is a feature of hemolytic anemia.

39
Q

A 32-year-old woman, accompanied by his brother, presents with complaint that every time she gets a cut, it takes a long for the bleeding to stop. She also mentions that she is distressed with her heavy periods, for which she has to use 10 pads a day. Her brother mentions that she has been like that since he can remember. She is otherwise healthy with no significant finding on physical examination. Which one of the following is the most likely diagnosis?
A. Hemophilia A.
B. Stuart disease.
C. Von Willebrand disease.
D. Factor IX deficiency.
E. Immune thrombocytopenic purpura (ITP).

A

**C. Von Willebrand disease. **

Long-standing history of prolonged bleeding after trauma and history of menorrhagea in an otherwise healthy woman is highly suggestive of Von Willebrand disease as the most likely cause.

Von Willebrand disease (VWD) is an inherited disease with many different types (22 types) and clinical pictures. Almost all types cause mild bleeding problems with excellent prognosis. The most common types often have an autosomal dominant inheritance. This disease is very common (1 in 100 population) and is the most common inherited bleeding disorder.

Von Willebrand factor is a circulating factor that is attached to factor VIII. This factor by gluing platelets together and to the vascular lining plays the earliest role in coagulation. Because the clinical disease can be very mild, most cases will never be diagnosed, but if symptomatic, the symptoms can include:
- Easy bruising
- Mucosal bleeding (e.g. epistaxis, menorrhagoea, gastrointestinal bleeding, etc)
- No history of hemarthroses or intramuscular hematomas (except for type 3 that is very rare but can have musculoskeletal manifestations)
- Prolonged bleeding after trauma or surgery

(Option A) Hemophilia A presents with factor type of bleeding (deep and delayed). As both haemophilia A and B are X-linked, women can all be carriers. Homozygous females never born.

(Option B) Stuart disease or factor X (Stuart-Prower factor) deficiency is one of the world’s rarest factor deficiencies. Factor X is a vitamin K-dependent factor that serves as the first enzyme in the common pathway of thrombus formation. It can cause both platelet and factor types of bleeding.

(Option D) Like hemophilia A, factor IX deficiency (haemophilia B) is never seen in an alive woman.

(Option E) ITP presents with petechiae and bruising rather than prolonged bleeding. Furthermore, compared to VWD, chronic ITP in adults is less common.

40
Q

A 32-year-old African male presents to the Emergency Department with fatigue, jaundice, dark-colored urine and acute decrease in hemoglobin for the past four days. Liver function tests are normal, except for an elevated unconjugated bilirubin level. Which one of the laboratory results would be most consistent with the diagnosis of hemolysis due to glucose-6-phosphate dehydrogenase deficiency?
A. Spherocytosis.
B. Schistocytosis.
C. Positive Coomb’s test.
D. Heinz bodies.
E. Elliptocytosis.

A

**D. Heinz bodies. **

Glucose-6-phosphate dehydrogenase (G6PD) deficiency is the most common enzyme deficiency in human, affecting 400 million people worldwide, with a high prevalence in people of African, Asian, and Mediterranean descent. The condition is inherited as an X-linked recessive disorder. It is polymorphic with more than 300 variants.

G6PD deficiency can present as neonatal hyperbilirubinemia. People with this disorder can experience episodes of brisk hemolysis in response to oxidative stresses or, less commonly, have chronic hemolysis. However, many individuals with G6PD deficiency are asymptomatic.
G6PD deficiency confers partial protection against malaria.

The diagnosis of G6PD deficiency can be made on the basis of a well-documented history, evidence of hemolysis, a peripheral-blood smear showing Heinz bodies (erythrocytes with denatured hemoglobin) and ‘bite cells’. Measurement of G6PD activity while the patient is in remission is the gold-standard diagnostic test for G6PD deficiency.

G6PD deficiency should be suspected in all patients with non-immune acute hemolysis and no spherocytosis on laboratory testing.

(Option A) Spherocytes are characteristic feature in hereditary spherocytosis.

(Option B) Schistocytes are seen in microangiopathic hemolytic anemia (MAHA) associated with hemolytic uremic syndrome and thrombotic thrombocytopenic purpura (TTP).

(Option C) Positive Coombs test is the characteristic feature of immune-mediates hemolytic anemia.

(Option E) Elliptocytosis, also known and ovalocytosis, is a feature of hereditary elliptocytosis – an inherited disorder in which an abnormally large number of patient’s red blood cells are elliptical rather than the typical biconcave disc-shaped.

41
Q

Which one of the following is the most common cause of anemia in geriatric population?
A. Iron deficiency anemia from blood loss.
B. Nutritional anemia from vitamin B12 deficiency.
C. Anemia of chronic disease.
D. Hemolytic anemia.
E. Myelodysplastic anemia.

A

**C. Anemia of chronic disease. **

Chronic diseases are the most common causes of anemia in the geriatric population accounting for 30-45% of cases. Iron deficiency is the second most common cause.

Cause: Percentage of cases
Chronic diseases 30-45%
Iron deficiency 15-30%
Post-hemorrhagic 5-10%
Vitamin B12/B9 deficiency 5-10% Chronic leukemia and lymphoma 5% Myelodysplastic syndromes 5%
No identifiable cause 15-25%

42
Q

A 70-year-old man from a low-level-of-care nursing home is brought to the hospital after he had a fall 3 hours ago. He is on warfarin for atrial fibrillation (AF). Laboratory studies show that he has an INR of 4.9. A CT scan of the head is ordered which is normal. Other investigations are unremarkable. Which one of the following is the next best step in management?
A. Stop warfarin.
B. Skip one dose of warfarin.
C. Give fresh frozen plasma (FFP).
D. Give vitamin K.
E. Repeat CT scan of the head in 2 days.

A

**B. Skip one dose of warfarin. **

The therapeutic range of warfarin for most patients varies between 2 to 3.5 (2-3, or occasionally 2.5-3.5).

Increased INR beyond therapeutic levels are associated with higher risk of bleeding. For patients with an INR above the therapeutic range but less than 5, who are not bleeding, skipping the next dose of warfarin and resuming lower doses once the INR is within the therapeutic range is the recommended management.

Ceasing warfarin, FFP, vitamin K (intravenously) and Prothrombinex are used in situations where there is active bleeding or the risk of bleeding is high.

43
Q

A group of college students are admitted to the emergency department with bruises over their skin, dark-colored urine and shock one week after they were back from camping in a forest. Which one of the following is most likely to be the cause of this presentation?
A. Giardia lamblia.
B. Escherichia coli.
C. Disseminated intravascular coagulopathy.
D. Staphylococcus aureus.
E. Ross River fever.

A

**B. Escherichia coli. **

The bruises over the skin (petechial rash), kidney involvement, and mental status alteration are consistent with thrombotic thrombocytopenic purpura (TTP) as the most likely diagnosis.

TTP is a rare blood disorder characterized by clotting in small blood vessels resulting in a low platelet count. In its full blown form, the disease consists of the pentad of (1) microangiopathic hemolytic anemia,
(2) thrombocytopenic purpura,
(3) neurological abnormalities,
(4) fever (non-infectious), and
(5) renal disease.
TTP can affect any organ system, but involvement of the peripheral blood, the central nervous system, and the kidneys causes the clinical manifestation.

Neurological manifestations include alteration in mental status, seizures, hemiplegia, paresthesias, visual disturbances, and aphasia. Patients may notice dark urine from hemoglobinuria. Severe bleeding from thrombocytopenia is unusual, but petechiae are common. Fever may occur in 50% of patients

E-coli O157:H7, E-coli O104:H4 and some other Shiga toxin-producing bacteria are the most common cause in children with TTP and hemolytic uraemic syndrome (HUS). In adults, many cases are idiopathic, but the same bacteria can cause these diseases as well. In adults immunosuppression and pregnancy can be other possible causes.

Since a group of persons are affected simultaneously, it is very likely that E-coli has been the etiology of the TTP.

(Option A) Giardia lamblia infection manifest with a different clinical picture including diarrhoea (non-bloody), flatulence, and crampy abdominal pain.

(Option C) DIC can present with similar picture (bleeding, shock, renal dysfunction, hepatic dysfunction, and central nervous system problems); however, the coincidence makes E-coli infection a more likely diagnosis.

(Option D) Staphylococcus aureus causes acute gastroenteritis with vomiting (more prominent) and diarrhea. Petechiae, neurological manifestations and shock are not present.

(Option E) Ross River fever is a mosquito-borne disease presenting with fever, rash, and polyarhtralgia.